Download Content Covered by the ACT English Test - Raceland

Document related concepts

Spanish grammar wikipedia , lookup

Malay grammar wikipedia , lookup

Transcript
English Test Description
English | Mathematics | Reading | Science | Writing
The English test is a 75-question, 45-minute test, covering:
Usage/Mechanics
Rhetorical Skills



punctuation
grammar and usage
sentence structure


strategy
organization

style
Read more about English Test content.
Spelling, vocabulary, and rote recall of rules of grammar aren't tested. See sample questions or read tips and
strategies.
The test consists of five prose passages, each one accompanied by multiple-choice test questions. Different passage
types are included to provide variety.
Some questions refer to underlined portions of the passage and offer several alternatives to the underlined portion.
You must decide which choice is most appropriate in the context of the passage.
Some questions ask about an underlined portion, a section of the passage, or the passage as a whole. You must
decide which choice best answers the question posed.
Many questions include "NO CHANGE" to the underlined portion or the passage as one of the choices.
The questions are numbered consecutively. Each question number corresponds to an underlined portion in the
passage or to a box located in the passage.
Content Covered by the ACT English Test
Six elements of effective writing are included in the English Test: punctuation, grammar and usage, sentence
structure, strategy, organization, and style. The questions covering punctuation, grammar, and sentence structure
make up the Usage/Mechanics subscore. The questions covering strategy, organization, and style make up the
Rhetorical Skills subscore.
Usage/Mechanics
 Punctuation (13%). Questions in this category test your knowledge of the conventions of internal and endof-sentence punctuation, with emphasis on the relationship of punctuation to meaning (for example,
avoiding ambiguity, indicating appositives).
 Grammar and Usage (16%). Questions in this category test your understanding of agreement between
subject and verb, between pronoun and antecedent, and between modifiers and the word modified; verb
formation; pronoun case; formation of comparative and superlative adjectives and adverbs; and idiomatic
usage.
 Sentence Structure (24%). Questions in this category test your understanding of relationships between
and among clauses, placement of modifiers, and shifts in construction.
Rhetorical Skills
 Strategy (16%). Questions in this category test how well you develop a given topic by choosing expressions
appropriate to an essay's audience and purpose; judging the effect of adding, revising, or deleting
supporting material; and judging the relevance of statements in context.
 Organization (15%). Questions in this category test how well you organize ideas and choose effective
opening, transitional, and closing sentences.
 Style (16%). Questions in this category test how well you select precise and appropriate words and images,
maintain the level of style and tone in an essay, manage sentence elements for rhetorical effectiveness, and
avoid ambiguous pronoun references, wordiness, and redundancy.
Tips for the ACT Multiple-choice Tests
English
 Be aware of the writing style used in each passage.
 Consider the elements of writing that are included in each underlined portion of the passage. Some questions
will ask you to base your decision on some specific element of writing, such as the tone or emphasis the
text should convey.
 Be aware of questions with no underlined portions—that means you will be asked about a section of the
passage or about the passage as a whole.
 Examine each answer choice and determine how it differs from the others. Many of the questions in the test
will involve more than one aspect of writing.
 Read and consider all of the answer choices before you choose the one that best responds to the question.
 Determine the best answer.
 Reread the sentence, using your selected answer.
Mathematics
 Read each question carefully to make sure you understand the type of answer required.
 If you choose to use a calculator, be sure it is permitted, is working on test day, and has reliable batteries.





Use your calculator wisely.
Solve the problem.
Locate your solution among the answer choices.
Make sure you answer the question asked.
Make sure your answer is reasonable.
Check your work.
Reading
 Read the passage carefully.
 Read and consider all of the answer choices before you choose the one that best responds to the question.
 Refer to the passage when answering the questions.
Science




Read the passage carefully.
Refer to the scientific information in the passage when answering the question.
Read and consider all of the answer choices before you choose the one that best responds to the question.
Note conflicting viewpoints in some passages.
http://www.sparknotes.com/testprep/books/act/chapter2.rhtml
General Strategies for Taking the ACT
IMAGINE TWO CHILDREN PLAYING TAG IN the forest. Who will win—the girl who never stumbles because she
knows the placement of every tree and all the twists and turns and hiding spots, or the kid who keeps falling down
and tripping over roots because he doesn’t pay any attention to the landscape? The answer is obvious. Even if the
other kid is faster and more athletic, the girl will win because she knows how to navigate the territory and use it to
her advantage.
This example of tag in the forest is extreme, but it illustrates a point. The structure of the ACT is the forest. Taking
the test is the game of tag. And no one likes to lose at tag.
In this chapter, you’ll learn how to take advantage of the ACT’s structure to achieve the score you want. You’ll learn
basic rules for taking the ACT, as well as pacing and preparation strategies. These are the general test-taking
strategies that you should use in all sections of the test. There are, of course, specific strategies for each of the
individual Subject Tests. We’ll cover those in the chapters devoted to each Subject Test.
Seven Basic Rules for Taking the ACT
These seven rules apply to every section of the ACT. They really are just commonsense guidelines, but it’s amazing
how the pressure and time constraints of the ACT can warp and mangle common sense. We list them here because
you should always have these rules of test taking resting gently in your mind as you take the test. You don’t need to
focus on them obsessively, but you should be sure not to forget them. They will help you save time and cut down on
careless errors.
1. Know the Instructions for Each Subject Test
Since you’ll need all the time you can get, don’t waste time reading the Subject Test instructions during the actual
test. Learn the instructions beforehand by taking practice tests and reading our chapters on the Subject Tests.
2. Use Your Test Booklet as Scratch Paper
Some students seem to think their test booklet has to look “pretty” at the end of the test. Don’t be one of those
students. A pristine test booklet is a sad test booklet. In the Math Test, the ACT writers even give you “figuring”
space for drawing diagrams and writing out solutions. You should write down all your work for math problems, in
case you want to return to them later to complete the question or check your answer. The Math Test isn’t the only
place where you can benefit from marginal scribbling, though. Making margin notes alongside the Reading and
Science Reasoning passages can help you stay on track when answering the subsequent questions. In addition, if you
want to skip a question and come back to it later, you should make a distinctive mark next to it, so you won’t miss it
on your second pass through the questions.
3. Answer Easy Questions before Hard Questions
This is a crucial strategy for the ACT. Since all questions within a Subject Test are worth the same number of points,
there’s no point slaving away over a difficult question if doing so requires several minutes. In the same amount of
time, you probably could have racked up points by answering a bunch of easy, less time-consuming questions.
In summary, answer the easy and moderate questions first. That way you’ll make sure that you get to see all the
questions on the test that you have a good shot of getting right, while saving the leftover time for the difficult
questions.
4. Don’t Get Bogged Down by a Hard Question
This rule may seem obvious, but many people have a hard time letting go of a question. If you’ve spent a significant
amount of time on a problem (in ACT world, a minute and a half is a lot of time) and haven’t gotten close to
answering it, just let it go. Leaving a question unfinished may seem like giving up or wasting time you’ve already
spent, but you can come back to the problem after you’ve answered the easy ones. The time you spent on the
problem earlier won’t be wasted. When you come back to the problem, you’ll already have done part of the work
needed to solve it.
This strategy goes hand in hand with Rule 3. After all, the tough question that’s chewing up your time isn’t worth
more to the computer grading your answer sheet than the easy questions nearby.
5. Avoid Carelessness
There are two kinds of carelessness that threaten you as an ACT test taker. The first kind is obvious: making
mistakes because you are moving too quickly through the questions. Speeding through the test can result in
misinterpreting a question or missing a crucial piece of information. You should always be aware of this kind of error
because the ACT writers have written the test with speedy test takers in mind: they often include tempting “partial
answers” among the answer choices. A partial answer is the result of some, but not all, of the steps needed to solve a
problem. If you rush through a question, you may mistake a partial answer for the real answer. Students often fall
into the speeding trap when they become confused, since confusion brings nervousness and fear of falling behind.
But those moments of confusion are precisely the moments when you should take a second to slow down. Take a
deep breath, look at the question, and make a sober decision about whether or not you can answer it. If you can, dive
back in. If you can’t, skip the question and go on to the next one.
The second kind of carelessness arises from frustration or lack of confidence. Don’t allow yourself to assume a
defeatist attitude toward questions that appear to be complex. While some of these questions may actually be
complex, some of them will be fairly simple questions disguised in complex-sounding terms. You should at least
skim every question to see whether you have a feasible chance of answering it. Assuming you can’t answer a question
is like returning a present you’ve never even opened.
6. Be Careful Bubbling In Your Answers
Imagine this: you get all the right answers to the ACT questions, but you fill in all the wrong bubbles. The scoring
computer doesn’t care that you did the right work; all it cares about are the blackened bubbles on the answer sheet,
and the wrong answers that they indicate.
Protect yourself against this terrifying possibility with careful bubbling. An easy way to prevent slips on the ACT
answer sheet is to pay attention to the letters being bubbled. Odd-numbered answers are lettered A, B, C, D (except
on the Math Test, where they are A, B, C, D, E), and even-numbered answers are lettered F, G, H, J (except on the
Math Test, where they are F, G, H, J, K).
You may also want to try bubbling in groups (five at a time or a page at a time) rather than answering one by one.
Circle the answers in the test booklet as you go through the page, and then transfer the answers over to the answer
sheet as a group. This method should increase your speed and accuracy in filling out the answer sheet. To further
increase your accuracy, say the question number and the answer in your head as you fill out the grid: “Number 24, F.
Number 25, C. Number 26, J.”
7. Always Guess When You Don’t Know the Answer
We will discuss guessing below in “The Meaning of Multiple Choice,” but the basic rule is: always guess! You’re
much better off guessing than leaving an answer blank because there is no penalty for wrong answers.
The Meaning of Multiple Choice
As we’ve suggested throughout this chapter, the multiple-choice format of the ACT should affect the way you
approach the questions. In this section, we’ll discuss exactly how.
Only the Answer Matters
A machine, not a person, will score your test. This scoring machine does not care how you came to your answers; it
cares only whether your answers are correct and readable in little oval form. The test booklet in which you worked
out your answers gets thrown in the garbage, or, if your proctor is conscientious, into a recycling bin.
On the ACT, no one looks at your work. If you get a question right, it doesn’t matter whether you did impeccable
work. In fact, it doesn’t even matter whether you knew the answer or guessed. The multiple-choice structure of the
test is a message to you from the ACT: “We only care about your answers.” Remember, the ACT is your tool to get
into college, so treat it as a tool. It wants right answers? Give it right answers, as many as possible, using whatever
strategies you can.
Multiple Choice: You’ve Already Got the Answers
When you look at any ACT multiple-choice question, the answer is already right there in front of you. Of course, the
ACT writers don’t just give you the correct answer; they hide it among a bunch of incorrect answer choices. Your job
on each question is to find the right answer. Because the answer is right there, begging to be found, you have two
methods you can use to try to get the correct answer:
1. Look through the answer choices and pick out the one that is correct.
2. Look at the answer choices and eliminate wrong answers until there’s only one answer left.
Both methods have their advantages: you are better off using one in some situations and the other in others. In a
perfect scenario in which you are sure how to answer a question, finding the right answer immediately is clearly
better than chipping away at the wrong answers. Coming to a conclusion about a problem and then picking the
single correct choice is a much simpler and quicker process than going through every answer choice and discarding
the four that are wrong.
However, when you are unsure how to solve the problem, eliminating wrong answers becomes more attractive and
appropriate. By focusing on the answers to problems that are giving you trouble, you might be able to use the answer
choices to lead you in the right direction, or to solve the problem through trial and error. You also might be able to
eliminate answer choices through a variety of strategies (these strategies vary by question type; we’ll cover them in
the chapters dedicated to each type of question). In some cases, you might be able to eliminate all the wrong
answers. In others, you might only be able to eliminate one, which will still improve your odds when you attempt to
guess.
Part of your preparation for the ACT should be to get some sense of when to use each strategy. Using the right
strategy can increase your speed without affecting your accuracy, giving you more time to work on and answer as
many questions as possible.
Guessing and the ACT
We’ve said it once, but it’s important enough to bear repetition: whenever you can’t answer a question on the ACT,
you must guess. You are not penalized for getting a question wrong, so guessing can only help your score.
Random Guessing and Educated Guessing
There are actually two kinds of guesses: random and educated. Random guesser Charlie Franklin will always guess C
or F because he really, really likes those letters. Using this method, Charlie has a pretty good chance of getting about
25 percent of the questions right, yielding a Composite Score of about 11. That’s not too shabby, considering Charlie
expended practically no intellectual energy beyond identifying C and F as the first letters of his first and last names.
But what about educated guesser Celia? Instead of immediately guessing on each question, she works to eliminate
answers, always getting rid of two choices for each question. She then guesses between the remaining choices and
has a 50 percent chance of getting the correct answer. Celia will therefore get about half of the questions on the test
correct. Her Composite Score will be about a 19, which is an average score on the ACT.
The example of these two guessers should show you that while blind guessing can help you, educated guessing can
really help you. For example, let’s say you know the correct answer for half of the questions and you guess randomly
on the remaining half. Your score will probably be a 22—three points higher than the score you’d get leaving half of
the answers blank. Now let’s say you know the correct answer for half of the questions and you make educated
guesses on the remaining half, narrowing the choices to two. You can probably score a 26 with this method, landing
you in the 90th percentile of test takers. This is a good score, and to get it you only need to be certain of half the
answers.
“Always guess” really means “always eliminate as many answer choices as possible and then guess.”
A Note to the Timid Guesser
Some students feel that guessing is like cheating. They believe that by guessing, they are getting points they don’t
really deserve. Such a belief might be noble, but it is also mistaken, for two reasons.
First, educated guessing is actually a form of partial credit on the ACT. Let’s say you’re taking the ACT and come
upon a question you can’t quite figure out. Yet while you aren’t sure of the definite answer, you are sure that two of
the answer choices can’t be right. In other words, you can eliminate two of the four answer choices, leaving you with
a one in two chance of guessing correctly between the remaining two answer choices. Now let’s say someone else is
taking the same test and gets to the same question. But this person is completely flummoxed. He can’t eliminate any
answer choices. When this person guesses, he has only a one in four chance of guessing correctly. Your extra
knowledge, which allowed you to eliminate some answer choices, gives you better odds of getting this question right,
exactly as extra knowledge should.
Second, the people who made the ACT thought very hard about how the scoring of the test should work. When they
decided that they wouldn’t include a penalty for wrong answers, they knew that the lack of a penalty would allow
people to guess. In other words, they built the test with the specific understanding that people would guess on every
question they couldn’t answer. The test wants you to guess. So go ahead and do it.
Pacing
The ACT presents you with a ton of questions and, despite its three-hour length, not that much time to answer them.
As you take the test, you will probably feel some pressure to answer quickly. As we’ve already discussed, getting
bogged down on a single question is not a good thing. But rushing isn’t any good either. In the end, there’s no real
difference between answering very few questions and answering lots of questions incorrectly: both will lead to low
scores. What you have to do is find a happy medium, a groove, a speed at which you can be both accurate and
efficient, and get the score you want. Finding this pace is a tricky task, but it will come through practice and strategy.
Setting a Target Score
The ACT is your tool to get into college. Therefore, a perfect score on the ACT is not a 36, it’s the score that gets you
into the colleges of your choice. Once you set a target score, your efforts should be directed toward achieving that
score and not necessarily a 36.
In setting a target score, the first rule is to be honest and realistic. Base your target score on the schools you want to
attend, and use the results from your practice tests to decide what’s realistic. If you score a 20 on your first practice
test, your target score probably should not be a 30. Instead, aim for a 23 or 24. Your scores will likely increase on
your second test simply because you’ll be more experienced than you were the first time, and then you can work on
getting several extra problems right on each Subject Test.
Your Target Score Determines Your Strategy and Pace
Your target score should affect your overall approach to the test. Cathy, whose target score is 31, is going to use a
different strategy and pace from Elvie, whose target score is 20. Cathy must work quickly without becoming careless
to get 90 percent of her questions right. Elvie, on the other hand, can afford to work more slowly; to get a 20, she
needs to answer approximately half of the questions correctly. Elvie can focus her energy on carefully answering
about 60 percent of the questions, allowing for some wrong answers; then she can guess on the remaining questions.
Cathy needs to focus on every question to get her 90 percent. Also allowing for some wrong answers, she should aim
to answer all the questions correctly.
Of course, this is all a bit like the chicken and the egg conundrum. Cathy’s target score is probably higher than
Elvie’s because she is a faster and better test taker than Elvie. Elvie needs the extra time to spend on each problem
because she is a slower worker than Cathy. It’s not as though Elvie generates a lot of extra time in which she can
doodle or draw elaborate diagrams by concentrating on a smaller number of questions. All of that extra time per
question is being put to use by Elvie because she needs it in order to get the right answer.
The point of this anecdote: Adjust your pacing to the score you want, but also be honest with yourself about what
pace you can maintain. The following charts will give you an idea of the number of questions you need to get right in
order to receive certain scaled scores on the ACT. Use these charts to determine the number of correct answers you
need in order to achieve your target score.
English
Math
Target Score
# Right
Target Score
# Right
36
75
36
60
30
69–70
30
53–54
26
60–62
26
44–45
23
52–54
23
38–39
20
44–46
20
32–33
17
36–38
17
23–25
11
19–21
11
7–8
Reading
Science Reasoning
Target Score
# Right
Target Score
# Right
36
40
36
40
30
35
30
37
26
30–31
26
32–33
23
26–27
23
27–28
20
22
20
22–23
17
18
17
16–17
11
9–10
11
7
The first target score you set doesn’t have to be your last. If you reach your initial target score, set a new, higher
score and try increasing the pace at which you work. In setting preparatory target scores, focus on improving a
couple points at a time. In the end, incremental change will work better than a giant leap.
The White Rabbit Syndrome: Watching the Clock
Because the ACT is a timed test, you should always be aware of the time. The proctor at the test center will strictly
enforce the time limits for each Subject Test. Even if you have only one question left to answer, you won’t be allowed
to fill in that bubble.
As you take the test, watch the clock. You shouldn’t be checking it every two minutes, since you will only waste time
and give yourself a headache. But you should check occasionally to make sure you are on pace to achieve your target
score. If you’re Cathy, aiming to answer 90 percent of the questions correctly, you’ll be in trouble if you’ve answered
only 40 of the 75 English questions in 30 minutes (the English Test is 45 minutes long). If you’re Elvie, aiming for 60
percent of the questions, answering 40 English questions in 30 minutes is a pretty good pace.
Preparing for the ACT
Preparation is the key to success on the ACT. When the ACT is lurking sometime far in the future, it can be difficult
to motivate yourself to study. Establishing an organized study routine can help keep you on track as you approach
the test date.
Setting Up a Study Schedule
Rather than simply telling yourself to study each week, you might want to write down an actual schedule, just as you
have a schedule of classes at school. Keep this schedule where you’ll see it every day, and consider showing it to a
parent who will nag you incessantly when you don’t follow it. (You might as well use your parents’ nagging
capabilities to your own advantage for once.) You should reward yourself for keeping to your schedule.
You should allot at least a few hours a week to studying, depending on how much time you have before the test date.
If you start preparing five weeks in advance, you might consider studying one subject per week, with the last week
left over for light review. Our chapters on the individual tests will give you a solid review of the material you need to
know.
To complement your studying, take at least part of a practice test each week. We’ve given you two practice tests at
the back of this book. You don’t necessarily have to take a full practice test each week, but, if you’re preparing for
English one week, take a practice English Test to help focus your studying. We explain how practice tests can
function as powerful study tools in the chapter called “Practice Tests Are Your Best Friends.”
The Day of the Test
You must bring the following items to the test center on the day of the test:
1. Your admission ticket
2. Photo ID or a letter of identification
Unless a test proctor recognizes you, you will not be allowed in the test room without appropriate identification. We
also suggest that you bring the following:
1. Number Two pencils
2. A calculator. You should bring the calculator you normally use (preferably with an extra battery). You don’t
want to get stuck searching frantically for the right buttons on an unfamiliar calculator.
3. A watch. Your test room may not have a clock, or the clock may not be visible from where you’re sitting.
Since the test proctors only call out the time five minutes before the end of each section, you have to rely on
yourself to know how much time remains.
4. A snack, to keep up that energy.
5. Lucky clothes. Why not?
The ACT English Test
THE ACT ENGLISH TEST ASSESSES YOUR knowledge of English grammar and writing. On the test, you will have
45 minutes to answer 75 questions. That may seem like a large number of questions and relatively little time, but the
English Test, more than any other ACT Subject Test, assesses what you already know, rather than what you can
figure out if you are given certain information. Essentially, this means you can be completely prepared for the
English Test if you study all the material it covers. This section will teach you exactly that material.
Instructions for the English Test
You should learn the instructions for the English Test long before you arrive at the test center. In fact, your first step
in preparing for the ACT should be learning the instructions for all four Subject Tests. On the actual test, the Subject
Test instructions are time-consuming obstacles, which you can remove by learning them in advance. You can also
benefit from them while you study because they contain valuable information about ACT questions and how to
answer them.
At the start of each chapter on an individual Subject Test, we’ve given you a complete summary of the Subject Test
instructions. Read through each set of instructions several times until you know them all like the back of your hand.
The English Test instructions are particularly long, so you’ll save yourself time on the test by learning them now.
Instructions: There are five passages on this subject test. You should read each passage once before answering the questions
on it. In order to answer correctly, you may need to read several sentences beyond the question.
There are two question formats within the passages. In one format, you will find words and phrases that have been
underlined and assigned numbers. These numbers will correspond with sets of alternative words/phrases, given in the right -
hand column of the test booklet. From the sets of alternative s, choose the answer choice that works best in context, keeping
in mind whether it employs standard written English, whether it gets across the idea of the section, and whether it suits the
tone and style of the passage. You will usually be offered the opt ion “NO CHANGE,” which you should choose if you think
the version found in the passage is best.
In the second format, you will see boxed numbers referring to sections of the passage or to the passage as a whole. In
the right-hand column, you will be asked questions about or given alternatives for the sections marked by the boxes.
Choose the answer choice that best answers the question or completes the section.
After choosing your answer choice, fill in the corresponding bubble on the answer shee t.
These instructions will seem much clearer to you after you’ve seen the sample English Test questions in the
following sections.
The Format of the English Test
The five passages on the English Test contain two question formats: underlines and boxes. Both the underlines and
the boxes will be numbered so you can find the corresponding multiple-choice answers in the right-hand column of
the test booklet.
Below you’ll find a sample English Test paragraph, illustrating both question -formats:
[1] That summer my parents buy me my first
17. A.
17
NO CHANGE
B.
bought
C.
have bought
D.
buys
bike—my first true love. [2] One day, I crashed
into a tree and broke my leg. [3] Unfortunately,
my control of the bike was not as great as my
enthusiasm for it. [4] I spent all my afternoons
speeding around the neighborhood blocks.
18.
Which of the following
provides the most logical
ordering of the sentences in
Paragraph 3?
F.
3, 2, 1, 4
G.
3, 1, 4, 2
H.
1, 4, 3, 2
J.
1, 4, 2, 3
Question 17 demonstrates the underline format on the English Test. In this example, the word “buy” is underlined
and numbered 17, indicating that you can replace “buy” with answer choices B, C, or D, or keep it by selecting answer
choice A (“NO CHANGE”). Decide which answer choice makes the sentence grammatically correct, and fill in the
corresponding bubble on your answer sheet. (The correct answer is B.)
Question 18 is an example of a Rhetorical Skills question indicated by a boxed number. The boxed number indicates
that the question will deal with a large section of the passage, not just a few words. This question asks you to
reorganize the sentences of the paragraph in a logical manner. Once you’ve arrived at an answer, fill in the
corresponding bubble on your answer sheet. (The correct answer to this question is H.)
Don’t worry about the answers to these questions now. We’ll deal with the specific question types and the grammar
covered on the English Subject Test later on in this chapter.
The Content of the English Test
There are actually two types of content on the English Test: the content of the passages and the content of the
questions. Question content is the more important of the two.
When we say “the content of the passages,” we mean the subjects covered by the five English Test passages. The
passages usually cover a variety of subjects, ranging from historical discussions to personal narratives. Don’t worry
about passage content for now; it is important when answering certain Rhetorical Skills questions, which we’ll
discuss toward the end of this chapter, but the grammar of the passage is generally more important.
“The content of the questions” refers to the two kinds of material covered by the English Test: Usage/Mechanics and
Rhetorical Skills. The majority of this section is devoted to explaining the question content on the English Test. For
now, we’ll give you a brief summary of the material.
Usage/Mechanics Questions
The 40 Usage/Mechanics questions on the test deal with the proper use of standard written English. You can think
of them as the “technical” aspect of the test because they ask you to apply the rules of standard English to sections of
the passages. Questions covering usage and mechanics are almost always presented as underlined sections of the
passages. Usage/Mechanics questions test your understanding of the following categories:
1.
Punctuation (10 questions): Punctuation questions ask you to identify and correct any misplaced,
misused, or missing punctuation marks. The punctuation marks most commonly tested on the ACT are, in
order of decreasing frequency, commas, apostrophes, colons, and semicolons.
2. Basic Grammar and Usage (12 questions): Basic Grammar and Usage questions usually target a single
incorrect word that violates the conventional rules of English grammar. These questions frequently test your
knowledge of agreement issues and pronoun and verb forms and cases.
3. Sentence Structure (18 questions): Sentence Structure questions tend to deal with the sentence as a
whole. They test you on clause relationships, parallelism, and placement of modifiers.
If some of these Usage/Mechanics issues sound unfamiliar or confusing to you, don’t worry—later, in the
“Usage/Mechanics Questions on the English Test” chapter, we’ll review all of the material you need to know for
these questions.
Rhetorical Skills Questions
The 35 Rhetorical Skills questions test your ability to refine written English. If the Usage/Mechanics questions are
the technical aspect of the test, then the Rhetorical Skills questions are the intuitive aspect—but they require an
intuition you can develop through practice. The boxes you encounter on the test will deal with Rhetorical Skills
questions; some underlined sections may deal with Rhetorical Skills as well.
Rhetorical Skills questions break down into the following categories:
1.
Writing Strategy (12 questions): Writing Strategy questions are concerned with a passage’s
effectiveness. These questions require that you understand the point, purpose, and tone of a passage. When
answering these questions, you must decide the best way to support a point with evidence, to introduce and
conclude paragraphs, to make a transition between paragraphs, or to phrase a statement.
2. Organization (11 questions): Organization questions can deal with individual sentences, individual
paragraphs, or the passage as a whole. They will ask you either to restructure the passage or paragraph or to
decide on the best placement of a word or phrase within a sentence.
3. Style (12 questions): Style questions focus on effective word choice. They will ask you to eliminate
redundancy and to select the most appropriate word or phrase. In order to answer style questions correctly,
you need to understand the tone of a passage, and you need to have a good eye for clear written English.
Because Rhetorical Skills questions require a sense of good English writing, they tend to be more difficult than
Usage/Mechanics questions, which primarily require that you understand grammatical rules. This sense for good
writing can be developed through review and practice. You’ll have a chance for both in the “Rhetorical Skills
Questions on the English Test” chapter.
Memorization and the Content of the English Test
The ACT writers emphasize that the English Test is not a test of memorization. It would be more accurate to say that
the test does not explicitly test your memorization of rules of the English language.
You will not be tested on vocabulary on the English Test (unlike on the SAT Verbal, which is largely a vocabulary
memorization test), but having a decent vocabulary is important in answering style and strategy questions. The
questions often ask you to choose the most effective word or phrase. If you don’t know what some of the words
mean, you may not be able to make the right choice.
Technically, the test does not ask you to memorize grammar rules, but it should be obvious that doing well on the
test requires that you know the conventional rules of grammar. You won’t be asked to state the definition of a
gerund, but you’ll be in trouble if you can’t make your subjects and verbs agree or if you think a comma splice is
something tasty in your spice rack.
Obviously, you need to understand grammatical rules for the English Test. While knowing these rules does not
explicitly require memorization, most people begin to learn grammar by memorizing its rules.
Strategies for the English Test
ALTHOUGH THE ENGLISH TEST IS relatively straightforward, you should use certain strategies to improve your
speed and efficiency and to avoid any traps the ACT writers may have included. This section covers both broad
strategies for approaching the English Test as well as specific tips for eliminating multiple-choice answers.
Skim the Entire Passage before Answering the Questions
Don’t immediately jump to the questions. Instead, first read quickly through the passage you’re working on; then
begin answering the accompanying questions.
While reading the passage once through before getting to the questions may seem like extra work, it will prevent you
from making unnecessary errors. The English Test instructions warn that you may need to read beyond a question in
order to answer it correctly. By being familiar with the entire passage, you can avoid the problem of not having read
far enough ahead. Reading the entire passage will also help you with Rhetorical Skills questions by giving you an
understanding of the passage’s purpose, argument, and tone.
If you need further convincing, the following sample English Test question demonstrates why reading beyond the
underlined section is necessary:
her dogs has sleek, brown hair
14. F. No change
14
G.
are
H.
have
J. do not have
Seems pretty easy, doesn’t it? “Ah, a simple subject-verb agreement problem,” you’re probably thinking. “The
answer, obviously, is H.” But what if we show you the whole sentence?
The girl walking her dogs has sleek, brown hair
14. F. No change
14
G.
are
H.
have
J. do not have
Reading the rest of the sentence reveals that the sleek, brown hair belongs to a girl rather than a pack of dogs. The
question was about subject-verb agreement, but the words directly next to the underlined phrase misled you into
thinking that the subject was “her dogs” and not “the girl.” If you had read the passage first, you would have realized
that the correct answer is F.
Admittedly, this example exaggerates the case for reading beyond the question, but it gets our point across.
Ultimately, if you quickly read through the passage before tackling the questions, you’ll avoid unnecessary mistakes
without sacrificing much time.
Answer the Questions in the Order They Appear
Answer ACT English questions in the order in which they appear. This suggestion is really just common sense. After
all, the questions appear in a certain order for a reason: a question at the beginning deals with the beginning of the
passage, a question in the middle deals with the middle of the passage, and so on. An organization question in the
middle of a passage won’t ask you to reorganize the entire passage or a faraway section of the passage. It will ask you
to reorganize the material directly to the left of the question. Rhetorical Skills questions on the passage as a whole
appear at the end of the passage, and what better time to answer those questions that deal with the entire passage
than at the end?
Organization of Usage/Mechanics and Rhetorical Skills Questions
Questions on the English Test do not appear in order of difficulty. On many passages, you tend to see easy
Usage/Mechanics questions near the beginning and relatively difficult Rhetorical Skills questions at the end, but
there is no set rule about the order of their appearance.
Guess
If you come to a question you can’t answer, you can either draw a mark next to it so you can return later, or you can
guess right away, leaving the question behind forever. On the English Test, and only on the English Test, we suggest
guessing and moving on. As we stated earlier, the questions on the English Test assess what you already know rather
than what you can figure out, so if you don’t get the answer right off the bat (or a few seconds off the bat), you’re not
likely to get it by intense wriggling and head scratching. With that in mind, marking the question in order to come
back later seems like a needless waste of time—you might as well take your shot right away and move on to more
fruitful English territory.
Following this strategy, you should not move on to a new passage without answering all the questions from the
previous one. Needless to say, if you follow our suggestion and guess when you don’t know the answer, you won’t
encounter this problem. But if you do decide to return to a question you skipped, do so before moving on to the next
passage; otherwise, you’re likely to forget crucial details from the passage.
Eliminate Answer Choices
Educated guessing is always better than blind guessing. Whenever you guess, try first to eliminate some of the
multiple-choice answers to improve your odds of guessing correctly. Take a look at these sample answers:
A.
When I swung the bat I knew, I had hit a home run.
B.
When I swung the bat, I knew I had hit a home run.
C.
When I swing the bat I will know I always hit a home run.
D.
When, I swung the bat I knew, I had hit a home run.
You can probably figure out from these answer choices that there is a comma placement error. Choices A, B, and D
all give versions of the same sentence with different comma placement. Choice C, attempting to lure you off the right
track, offers a comma-less version of the sentence with nonsensically altered verb tenses.
Can you eliminate any of these answer choices? Well, choice C looks like a prime candidate for elimination because it
makes little sense. Choice D also looks like it can go because of the comma placed after “When,” which leaves the
word dangling at the beginning of the sentence. If you can eliminate either or both of these, you greatly increase the
chance that you’ll pick the correct answer, which is B.
Eliminating Answer Choices for Questions with Multiple Errors
Quite often, you will encounter questions that involve more than one error. While these questions may seem harder
to answer than single-error questions, you can benefit from the multiple errors when trying to eliminate answer
choices: if you can’t spot one error, you might spot the other.
Instead of tackling all the errors at once, you’ll have an easier time picking them off one by one. Let’s use the
following example:
A.
Cathys’ friends left they’re bags in the room.
B.
Cathy’s friends left there bags in the room.
C.
Cathys friends left their bags in the room.
D.
Cathy’s friends left their bags in the room.
These sentences contain two variations. If you focus on Cathy and her friends, you realize that you should eliminate
choices A and C for incorrect apostrophe placement. Now you’ve narrowed your options to B and D, which
respectively use “there” and “their” as possessive pronouns. If you don’t know the difference between the two, you
have a 50 percent chance of guessing the right answer. If you do know the difference (and you will, after you read the
Usage/Mechanics chapter), you know that “there bags” is incorrect and that the correct answer is therefore D.
Avoid Being Influenced by the Answer Choices
Be wary of answer choices that try to trick you into overcorrecting the problem. You shouldn’t be fooled into finding
additional “errors” by an answer choice that has completely made over the original. The correct answer to a question
is not necessarily the one that has changed the most elements of the underlined phrase.
Choose “NO CHANGE”
In fact, the correct answer to a question is not necessarily one that has changed anything at all. All Usage/Mechanics
questions and some Rhetorical Skills questions offer you “NO CHANGE” as an answer choice. Do not overlook “NO
CHANGE” as a possible answer to the problem. It is correct approximately 20 percent of the time it’s offered.
If your gut tells you there’s nothing wrong with the underlined phrase, don’t change the phrase.
If the Phrase Doesn’t Fit, You Must “OMIT”
You will often see the answer choice “OMIT the underlined portion.” By choosing it, you can remove the entire
underlined portion from the passage.
When an answer choice allows you to “OMIT the underlined portion,” think hard about that option. “OMIT,” when it
appears as an answer, is correct approximately 25 percent of the time. We don’t suggest that you go through the test
ticking off “OMIT” for every possible question, but we do want you to consider it as an answer.
“OMIT” is an attractive (and often correct) answer because it eliminates redundant or irrelevant statements. (For
more on redundancy, see the “Style” section under “Rhetorical Skills Questions on the English Test.”) For example,
The bag was free. I didn’t have to pay for it.
21
21. A.
B.
NO CHANGE
I paid five dollars for it.
C. I paid almost nothing for it.
D. Omit the underlined portion
The ACT writers want your edits to make the passage as concise as possible. A statement like the one above should
strike you as redundant because you clearly don’t need to pay for something that’s free—so why say the same thing
twice? If you choose choice A, you keep the redundant sentence in the passage and get the answer wrong. Choices B
and C don’t make much sense because they have you paying for the free bag. Choice D is the correct answer because
it omits an unnecessary statement. Without the second sentence, a reader still understands that the free bag didn’t
cost anything.
When deciding whether to omit, read the passage or sentence without the underlined portion and see whether the
new version of the sentence makes as much, if not more, sense to you as the original. If it does, go ahead and choose
“OMIT.” If the passage or sentence loses something in the omission, then turn to the other answer choices.
Usage/Mechanics Questions on the English Test
IN ORDER TO DO WELL ON THE ENGLISH Test, you need to know the basic rules of grammar. Specifically, you
need to know the rules of grammar most often tested by the ACT. This section will teach you the grammar you need
to know for the test. These Usage/Mechanics topics are:
1. Punctuation
2. Basic Grammar and Usage
3. Sentence Structure
Punctuation
Punctuation shows you how to read and understand sentences. For instance, the period at the end of the last
sentence indicated that the sentence had come to an end and that the next sentence would begin a new thought. We
could go on and on like this, but you get the point.
The ACT English Test requires that you know the rules for the following types of punctuation:
1.
2.
3.
4.
5.
6.
Commas
Apostrophes
Semicolons
Colons
Parentheses and Dashes
Periods, Question Marks, and Exclamation Points
Not all of these punctuation types are tested on every English Test. However, you can definitely expect to find
questions dealing with the first four items of the list on the English Test you take.
Commas
Misplaced, misused, and missing commas are the most frequent punctuation offenders on the English Test. Commas
can serve several functions within sentences:
Commas Separate Independent Clauses Joined by a Conjunction
An independent clause contains a subject and a verb (an independent clause can be as short as “I am” or “he read”),
and it can function as a sentence on its own. When you see a conjunction (and, but, for, or, nor, yet) joining
independent clauses, a comma should precede the conjunction. For example,
An independent clause contains a subject and a verb , and it can function as a sentence on its own.
Lesley wanted to sit outside, but it was raining.
Henry could tie the shoe himself, or he could ask Amanda to tie his shoe.
In each example, the clauses on both sides of the comma could stand as sentences on their own. With the addition of
the comma and conjunction, the two independent clauses become one sentence.
Commas Delineate a Series of Items
A series contains three or more items separated by commas. The items in a series can be either nouns (such as
“dog”) or verb phrases (such as “get in the car”). Commas are essentially the structural backbone of a series. For
example,
The hungry girl devoured a chicken, two pounds of pasta, and a chocolate cake.
When he learned his girlfriend was coming over, Nathaniel took a shower, brushed his teeth, and cleaned his room .
The comma follows all but the last item in the series. When using a conjunction, such as “and” or “or,” at the end of
the series, remember to precede it with a comma (“. . . brushed his teeth, and cleaned his room”).
Commas Separate Multiple Nonessential Adjectives Modifying a Noun
When two or more nonessential adjectives modify a noun, they should be separated by a comma. Of course, the key
to figuring out whether there should be a comma separating two adjectives is being able to determine whether the
adjectives are essential or nonessential. Luckily, there’s a simple rule that can help you: the order of nonessential
adjectives is interchangeable. For example,
Rebecca’s new dog has long, silky hair.
The loud, angry protesters mobbed the building.
These two sentences would make equal sense if you switched the order of the adjectives: “Rebecca’s new dog has
silky, long hair” and “The angry, loud protesters mobbed the building.”
The case is different if you have an essential adjective modifying the noun. Essential adjectives specify the nouns
they modify; they are bound to the noun, so that the noun loses meaning if separated from its adjective. A noun
modified by an essential adjective should be treated as a single noun. If you come across two adjectives modifying a
noun, and one is essential, you should not use a comma between them. For example,
My mother hates noisy electronic music.
“Electronic music” functions as an indivisible noun; “electronic” specifies the type of music the mother hates.
“Noisy” is a nonessential adjective modifying the noun “electronic music.” Changing the order of “noisy” and
“electronic” (“My mother hates electronic noisy music”) would not make sense. If you can’t change the order of two
adjectives preceding a noun, you know the adjective nearest the noun is essential, so you should not use a comma.
Commas Set Off Dependent Phrases and Clauses from the Main Clause of a Sentence
Unlike independent clauses, dependent phrases and clauses are not sentences in themselves; rather, they serve to
explain or embellish the main clause of a sentence. When they appear at the beginning of a sentence, they should be
set off from the main clause by a comma. For example,
Scared of monsters, Tina always checked under her bed before going to sleep.
After preparing an elaborate meal for herself, Anne was too tired to eat.
The first example shows a dependent clause (“Scared of monsters”) acting as an adjective modifying “Tina.” The
second example shows a dependent clause acting as an adverb. Since the adverbial clause is at the beginning of the
sentence, it needs to be set off from the main clause by a comma. Adverbial clauses should also be set off by commas
if they appear in the middle of a sentence. However, if an adverbial clause appears at the end of a sentence, you do
not need to use a comma. For example,
Anne was too tired to eat after preparing an elaborate meal for herself.
Commas Set Off Nonessential Phrases and Clauses
Nonessential phrases are like nonessential adjectives in that they embellish nouns without specifying them.
Nonessential phrases should be set off from the rest of the sentence by commas. For example,
Everyone voted Carrie, who is the most popular girl in our class, prom queen.
The decrepit street sign, which had stood in our town since 1799, finally fell down.
When you use nonessential phrases like the two above, you assume that “Carrie” and “the decrepit street sign” do
not need any further identification. If you remove the nonessential phrases, you should still be able to understand
the sentences.
Restrictive phrases, on the other hand, are not set off by commas because they are necessary to understand the
modified noun and the sentence as a whole. For example,
The girl who is sick missed three days of school.
The dog that ate the rotten steak fell down and died.
If you removed the restrictive phrases (“who is sick” and “that ate the rotten steak”) from these sentences, you would
be left wondering “which girl?” and “which dog?” These restrictive phrases are used to identify exactly which girl
missed school and exactly which dog died. Setting off “who is sick” in commas would assume that the girl’s identity
is never in doubt; there is only one girl who possibly could have missed school. In this case, we know the identity of
the girl only because the restrictive phrase specifies “the girl who is sick.”
Commas Set Off Appositives
Appositives are similar to nonessential phrases. An appositive is a phrase that renames or restates the modified
noun, usually enhancing it with additional information. For example,
Everyone voted Carrie, the most popular girl in school, prom queen.
The dog, a Yorkshire Terrier, barked at all the neighbors.
In these two examples, “the most popular girl in school” and “a Yorkshire Terrier” are appositives used to explain the
nouns they modify. You should be able to draw an imaginary equal sign between the noun and the appositive
modifying it: Carrie = the most popular girl in school, the dog = a Yorkshire Terrier. Because they are equal, you
should be able to swap them and retain the meaning of the sentence: “Everyone voted the most popular girl in
school, Carrie, prom queen.”
Apostrophes
Apostrophes are the second most commonly tested punctuation mark on the English Test. Apostrophes primarily
indicate possession, but they also take the place of omitted letters in contractions (for example, “was not” becomes
“wasn’t” and “it is” becomes “it’s”). You will be tested chiefly on your knowledge of the apostrophe’s possessive
function.
The Possessive and Singular Nouns
A singular noun (for example: Simon, the dog, the bottle) can be made possessive by adding an apostrophe followed
by an “s”. For example,
Simon’s teacher was in the room.
My mom forgot the dog’s food.
We removed the bottle’s label.
The apostrophe follows directly after the noun. If you move the apostrophe after the “s” (for example, if you write
“dogs’” rather than “dog’s”), you will change the meaning of the sentence (see “The Possessive and Plural Nouns”
below). If you forget the apostrophe altogether, you will render the sentence meaningless.
The Possessive and Plural Nouns
Most plural nouns (for example: the boys, the dogs, the bottles) can be made possessive by adding only an
apostrophe. For example,
The boys’ teacher was in the room.
My mom forgot the dogs’ food.
We removed the bottles’ labels.
The apostrophe directly follows plural nouns that end in “s” to make them possessive.
But for plural nouns that do not end in “s” (for example, “women”), you should treat the plural form as a singular
noun (i.e., add an apostrophe followed by an “s”). For example,
The women’s locker room needs to be cleaned.
The Possessive and Multiple Nouns
Sometimes you’ll want to indicate the possessive of more than one noun (Nick and Nora, Dan and Johann). The
placement of the apostrophe depends on whether the possessors share the possession. For example,
Nick and Nora’s dog solves crimes.
Dan’s and Johann’s socks are dirty.
In the example of Nick and Nora, the dog belongs to both of them, so you treat “Nick and Nora” as a single unit,
followed by a single apostrophe and “s.” In the second example, both Dan and Johann have dirty socks, but they
don’t share the same dirty socks, so you treat Dan and Johann as separate units, giving each an apostrophe and “s.”
The Possessive and Pronouns
Unlike nouns and proper nouns, the possessive case of pronouns does not use an apostrophe. The following chart
gives you nominative pronouns (the ones you use as subjects) and the corresponding possessive pronouns:
Nominative Pronoun
Possessive Pronoun
I
my
you (s.)
your
she
her
he
his
we
our
you (pl.)
your
they
their
it
its
who
whose
For example,
The dog chewed on its tail.
You should give him your wallet.
Don’t confuse the “its” and the “your” above with “it’s” and “you’re.” This mistake is frequently tested on the English
Test (see below).
ITS/IT’S, THEIR/THEY’RE
The ACT will test you on your ability to distinguish between “its” and “it’s.” “Its” is the possessive form of “it.” “It’s”
is the contraction of “it is.” This can be tricky to remember, since you are normally trained to associate apostrophes
with possession. But when you’re dealing with “its” versus “it’s,” the apostrophe signals a contraction. The same is
true for “their/they’re/there,” “your/you’re,” and “whose/who’s.” Make sure you are aware of these exceptions to the
apostrophe rule of possession.
Try the following practice problem:
Your face is red.
9
9. A.
NO CHANGE
B.
You’re face
C.
Your nose
D. OMIT the underlined portion.
You can eliminate choices C and D immediately: C changes the meaning of the sentence for no particular reason, and
D leaves you without a complete sentence. The decision comes down to “Your” and “You’re.” If you don’t know the
correct answer, try replacing “You’re” with “You are.” The resulting sentence is “You are face is red”—an odd remark.
The correct answer is A, “NO CHANGE.” You can employ this replacement technique whenever you don’t know the
answer to a possessive-or-contraction question. Once you replace the contraction with the full phrase, your ear will
tell you which choice is right.
Semicolons
You’ll usually find several questions dealing with semicolons on the English Test. The main functions of a semicolon
that you should know for the English Test are its ability to join related independent clauses and its use in a series.
The Semicolon and Two Independent Clauses
Semicolons are commonly used to separate two related but independent clauses. For example,
Julie ate five brownies; Eileen ate seven.
Josh needed to buy peas; he ran to the market.
In these cases, the semicolon functions as a “weak period.” It suggests a short pause before moving on to a related
thought, whereas a period suggests a full stop before moving on to a less-related thought. Generally, a period
between these independent clauses would work just as well as a semicolon, so the ACT won’t offer you a choice
between period or semicolon on the English Test. But you may see the semicolon employed as a weak period in an
answer choice; in that case, you should know that it is being used correctly.
Frequently, you will see two independent clauses joined by a semicolon and a transitional adverb (such as
consequently, however, furthermore, indeed, moreover, nevertheless, therefore, and thus). For example,
Julie ate five brownies; however, Eileen ate seven.
Josh needed to buy peas; thus he ran to the market.
These sentences function similarly to those joined by a comma and a conjunction. Here, the semicolon replaces the
comma, and the transitional adverb replaces the conjunction. Most transitional adverbs should be followed by a
comma, but for short adverbs such as “thus,” the comma should be omitted.
The Semicolon and the Series: When the Comma’s Already Taken
The semicolon replaces the comma as the structural backbone of a series if the items already contain commas. For
example,
The tennis tournament featured the surprise comeback player, Koch, who dropped out last year due to injuries ; the up-andcoming star Popp, who dominated the junior tour; and the current favorite, Farrington, who won five of the last six
tournaments.
If you used commas rather than semicolons in the above sentence, anyone reading the sentence would feel pretty
confused. The semicolons in this example function exactly as commas do in a series, but they allow you to avoid
overpopulating the sentence with commas.
Colons
You’ll probably be tested on your knowledge of colons a couple of times on the English Test. The ACT writers want to
be sure that you know how colons introduce lists, explanations, and quotations.
The Colon and Expectation
Colons are used after complete sentences to introduce related information that usually comes in the form of a list, an
explanation, or a quotation. When you see a colon, you should know to expect elaborating information. For example,
The wedding had all the elements to make it a classic : the elegant bride, the weeping mother, and the fainting bridesmaids .
In this example, the colon is used to introduce a list of classic wedding elements. Without the list following the colon,
the sentence can stand alone (“The wedding had all the elements to make it a classic”). By naming the classic
elements of a wedding, the list serves mainly to explain and expand upon the independent sentence that precedes it.
Check out this example of another way to use colons:
The wedding had all the elements to ma ke it a classic: the elegant bride beamed as her mother wept and as the bridesmaids
fainted.
Here, the clause following the colon also has an explanatory function. In this case, the colon joins two independent
clauses, but the clause following the colon is used to explain and expand the first.
Colons can also be used to introduce quotations. For example,
The mother’s exclamation best summed up the wedding : “If only the bridesmaids hadn’t fainted!”
Here, the colon is used to introduce the mother’s exclamation. Make sure the quotation following the colon is related
to the sentence.
Colon Problems
You should learn the following rules in order to avoid erroneous colon use on the English Test:
A COLON SHOULD ALWAYS BE PRECEDED BY AN INDEPENDENT CLAUSE.
WRONG: The ingredients I need to make a cake: flour, butter, sugar, and icing.
RIGHT: I need several ingredients to make a cake: flour, butter, sugar, and icing.
In the “WRONG” example, a sentence fragment precedes the list of items. The sentence should be reworked to create
an independent clause before the colon.
THERE SHOULD NEVER BE MORE THAN ONE COLON IN A SENTENCE.
WRONG: He brought many items on the camping trip: a tent, a sleeping bag, a full cooking set, warm clothes, and several
pairs of shoes: sneakers, boots, and sandals.
RIGHT: He brought many items on the camping trip: a tent, a sleeping bag, a full cooking set, warm clothes, sneakers,
boots, and sandals.
If you see a sentence that contains more than one colon, the sentence needs to be rephrased. Lists within lists or
explanations within explanations do not work in standard written English.
Other ACT Punctuation
The English Test rarely tests punctuation marks other than the ones listed above. But in the odd case that the test
writers do throw in some other punctuation errors, you should know what to expect. The ACT officially states that it
covers, in addition to the punctuation mentioned above, the following punctuation marks:
Parentheses and Dashes
Parentheses usually surround words or phrases that break a sentence’s train of thought but provide explanatory
information for it. For example,
Their road trip (which they made in a convertible) lasted three weeks and spanned fourteen states.
Similarly, parenthetical sentences can be inserted between other sentences, adding additional information to them
without diverting their flow. For example,
Their road trip lasted three weeks and spanned fourteen states. (The one they took two years ago lasted two weeks and
covered ten states.) When they got home, they were exhausted.
In this example, the parenthetical information about the previous road trip is interesting but not completely relevant
to the other sentences. Note that when an entire sentence is enclosed within parentheses, the period should be inside
them as well.
Dashes function similarly to parentheses. Dashes indicate either an abrupt break in thought or an insertion of
additional, explanatory information.
He walked so slowly—with his lame leg he couldn’t go much faster —that even his neighbor’s toddler eventually overtook
him.
I don’t have the heart to refuse a friend’s request for help —do you?
Periods, Question Marks, and Exclamation Points
These are the least common forms of punctuation tested by the ACT. The ACT writers probably realized that these
sentence enders are easier to grasp than other forms of punctuation because they basically each have only one
function:
The sentence ends here.
Does the sentence end here?
Hooray, the sentence ends here!
The period in the first example indicates that the sentence has ended. In the second example, the question mark
indicates that a question is being asked. The third example is an exclamatory statement marked by an exclamation
point. Exclamation points should be used sparingly to indicate statements made with great emotion (for example,
anger, excitement, or agitation).
Basic Grammar and Usage
As you’ve probably already gathered, the English Test will never explicitly ask you to name a grammatical error. But
in order to identify and fix errors, you should know what they are. While you’ll often be able to rely on your ear to
detect errors, many of the questions will ask you to fix phrases that are fine for spoken English but not for formal
written English.
In the following section, we’ll cover these grammar issues, which appear on the English Test:
1.
2.
3.
4.
5.
6.
7.
Subject-Verb Agreement
Pronoun-Antecedent Agreement
Pronoun Cases
Verb Tenses
Adverbs and Adjectives
Idioms
Comparative and Superlative Modifiers
Subject-Verb Agreement
Singular verbs must accompany singular subjects, and plural verbs must accompany plural subjects.
SINGULAR: The man wears four ties.
His favorite college is in Nebraska.
Matt, along with his friends, goes to Coney Island.
PLURAL: The men wear four ties each.
His favorite colleges are in Nebraska.
Matt and his friends go to Coney Island.
In the first example with Matt, the subject is singular because the phrase “along with his friends” is isolated in
commas. But in the second example with Matt, his friends join the action; the subject becomes “Matt and his
friends,” calling for the change to a plural verb.
Subject-verb agreement is a simple idea, but ACT writers will make it tricky. Often, they’ll put the subject at one end
of the sentence and the verb a mile away. Try the following example:
An audience of thousands of expectant
people who have come from afar to listen
to live music in an outdoor setting seem
17
17. A. NO CHANGE
B.
seems
C. have seemed
D.
to seem
terrifying to a nervous performer.
To solve this problem, cross out the junk in the middle that separates the subject, “an audience,” from the verb
“seem.” Remember that the subject of a sentence can never be part of a phrase that begins with “of.” You’re left with:
An audience seem terrifying to a nervous performer.
Now you can see what the verb should be:
An audience seems terrifying to a nervous performer.
So the correct answer is B. Double-check by eliminating choices C and D because they are grammatically incorrect
(and because they don’t make much sense in the sentence).
As long as you can isolate the subject and verb, handling subject-verb agreement is relatively simple. But certain
cases of subject-verb agreement can be tricky. The ACT writers like to test you on several of these difficult types of
subject-verb agreement.
Collective Nouns
Collective nouns (such as committee, family, group, number, and team) can be either singular or plural. The verb
depends on whether the collective noun is being treated as a single unit or as divided individuals. For example:
SINGULAR: The number of people living in Florida varies from year to year.
PLURAL: A number of people living in Florida wish they had voted for Gore.
SINGULAR: The committee decides on the annual program.
PLURAL: The committee have disagreed on the annual program.
You can often determine whether a collective noun is singular or plural by examining the article (“the” or “a”) that
precedes it. As in the first example, “The number” is generally singular, while “A number” is generally plural. This
difference is demonstrated in the first example above. “The number” of people in Florida is a single entity—even
though it comprises multiple individuals—so it takes a singular verb, “varies.” “A number” of people, on the other
hand, behave as multiple individuals—even though they wish for the same thing, they act independently of each
other—so these people require a plural verb, “wish.”
Looking to the article preceding a noun is a useful trick when deciding whether the noun is singular or plural, but it
doesn’t always work. In the second example, “The committee” can be both singular and plural. How the committee
behaves (do they act together or apart?) decides whether the verb is singular or plural. If the committee does
something as a unified whole (“decides on the annual program”), then the verb is singular. If the committee are
divided in their actions (“have disagreed on the annual program”), then the verb is plural.
Indefinite Pronouns
Indefinite pronouns refer to persons or things that have not been specified. Matching indefinite pronouns with the
correct verb form can be tricky because some indefinite pronouns that seem to be plural are in fact singular.
Questions dealing with singular indefinite pronouns are popular with ACT writers, so you’d be wise to memorize a
few of these pronouns now. The following indefinite pronouns are always singular, and they tend to appear on the
English Test:
Another
Everybody
Nobody
Anybody
Everyone
No one
Anyone
Everything
Somebody
Anything
Each
Someone
All the indefinite pronouns in the list above should be followed by singular verbs. For example,
Anyone over the age of 21 is eligible to vote in the United States.
Each has its own patch of grass.
If you’re used to thinking these pronouns take plural verbs, these sentences probably sound weird to you. Your best
bet is to memorize the list above (it’s not very long!) and to remember that those pronouns take singular verbs.
You should also be aware that not all indefinite pronouns are singular. Some (for example, all, any, none, and some)
can be either singular or plural depending on the context of the sentence. Other indefinite pronouns (for example,
both, few, many, and several) are always plural. The differences among these indefinite pronouns can be very
confusing; determining what’s right often requires an astute sense of proper English (or good memorization). If
you’re struggling to remember the different indefinite pronouns, take comfort in these two things:
1. The most commonly tested indefinite pronouns are the singular ones in the list we gave you.
2. You probably won’t come across more than a couple of indefinite pronouns on the English Test you take.
Compound Subjects
Most compound subjects (subjects joined by “and”) should be plural:
Kerry and Vanessa live in Nantucket.
The blue bike and the red wagon need repairs.
The reasoning behind this rule is fairly simple: you have multiple subjects, so you need a plural noun. Thus “Kerry
and Vanessa live” and the “bike and wagon need.”
“There Is” or “There Are”?
Whether to use “there is” or “there are” depends on the singularity or plurality of the noun that the phrase is
pointing out. If you have five grapes, you should say: “There are five grapes.” If you have a cat, you should say:
“There is a cat.” The “is” and the “are” in these sentences are the main verbs, so they must agree with the noun.
“Or” and “Nor”
If you have singular subjects joined by an “or” or “nor,” the sentence always takes a singular verb. For example,
Either Susannah or Caitlin is going to be in trouble.
If one of the subjects is plural and the other is singular, the verb agrees with the subject closer to it. For example,
Neither the van nor the buses were operating today.
Either the dogs or the cat is responsible for the mess.
Both of these examples contain a singular and a plural subject. The main verb of the sentence is determined by the
subject nearest it: in the first example, “buses” is closer to the verb, so the verb is plural, and in the second example,
“cat” is closer to the verb, so the verb is singular.
Mathematics, News, Dollars, Physics
These and other words look plural but are singular in usage:
Today’s news was full of tragic stories.
Trust your gut instinct with these words. You’ll probably know they’re singular from everyday usage. “Dollars” is an
exceptional case—it’s singular when you’re talking about an amount of money (“ninety dollars is a big chunk of
change”) but plural when you’re discussing a particular group of bills (“the dollars in my pocket are green”).
Pronoun-Antecedent Agreement
The ACT writers usually include several pronoun-antecedent agreement errors on the English Test. An antecedent is
a word to which a later pronoun refers back. For example, in the sentence “Richard put on his shoes,” “Richard” is
the antecedent to which “his” refers. When the pronoun does not agree in gender or number with its antecedent,
there’s an agreement error. For example:
WRONG: Already late for the show, Mary couldn’t find their keys.
RIGHT: Already late for the show, Mary couldn’t find her keys.
Unless another sentence states that the keys belong to other people, the possessive pronoun should agree in gender
and number with “Mary.” As far as we can tell, Mary is a singular, feminine noun, so the pronoun should be too.
The example of Mary contained a fairly obvious example of incorrect agreement, but sometimes the agreement error
isn’t as obvious on the ACT. In everyday speech, we tend to say “someone lost their shoe” (wrong) rather than
“someone lost his shoe” (correct) or “someone lost her shoe” (also correct) because we don’t want to exclude either
gender and because “someone lost his or her shoe” sounds cumbersome. The common solution? We attempt gender
neutrality and brevity by using “their” instead of “his” or “her.” In informal speech, such a slip is okay. But if you see
it on the test, it’s an error.
You will also run into agreement errors where the antecedent is unclear. In these cases, the pronoun is ambiguous.
We use ambiguous pronouns all the time in everyday speech, but on the test (you guessed it) they’re wrong.
WRONG: Trot told Ted that he should get the mauve pants from the sale rack.
This sentence is wrong because we don’t know to whom “he” refers. Should Ted get the pants, or should Trot? Or
should neither, because mauve pants are never a good idea? You should restate the original sentence so all the
pertinent information is relayed without confusion or multiple meanings, such as “Trot told Ted that Ted should get
the pants….”
Pronoun Cases
The ACT writers will definitely include some questions on pronoun cases. Pronoun case refers to the role of the
pronoun in a sentence. There are three cases: nominative, objective, and possessive. You don’t need to know the
names of these cases, but you do need to know the differences between them (and knowing the names doesn’t hurt).
Here, we’ll briefly describe each case.
The Nominative Case
The nominative case should be used when a pronoun is the subject of a sentence—for example, “I went to the store”
and “They walked to the park.” You should also use a nominative pronoun after any form of to be:
WRONG: It was me on the phone.
RIGHT: It was I on the phone.
The right sentence may sound awkward to you, but it’s the correct use of the nominative. The people who laid down
the rules of grammar considered to be a grammatical equal sign, so when you have a sentence like “It was I on the
phone,” you should be able to do this: “It” = “I.” If that equation holds true, “I” should be able to take the place of “It”
in the sentence: “I was on the phone.”
PRONOUN COMPARISONS
The nominative also follows comparative clauses that usually begin with “as” or “than.” When a pronoun is involved
in a comparison, it must match the case of the other pronoun involved. For example,
WRONG: I’m fatter than her, so I’ll probably win this sumo wrestling match.
RIGHT: I’m fatter than she, so I’ll probably win this sumo wrestling match.
In this sentence, “I” is being compared to “her.” Obviously, these two pronouns are in different cases, so one of them
must be wrong. Since only “her” is in question, it must be wrong, and therefore “she” is the correct answer.
Another way to approach comparisons is to realize that comparisons usually omit words. For example, it’s
grammatically correct to say, “Alexis is stronger than Bill,” but that’s an abbreviated version of what you’re really
saying. The long version is, “Alexis is stronger than Bill is.” That last “is” is invisible in the abbreviated version, but
you must remember that it’s there. Now let’s go back to the sumo sentence. As in our Alexis and Bill example, we
don’t see the word “is” in the comparison, but it’s implied. If you see a comparison using a pronoun and you’re not
sure if the pronoun is correct, add the implied “is.” In this case, adding “is” leaves us with “I’m fatter than her is.”
That sounds wrong, so we know that “she” is the correct pronoun in this case.
The Objective Case
As may be obvious from its name, the objective case should be used when the pronoun is the object of another part
of speech, usually a preposition or a transitive verb (a verb that takes a direct object):
PREPOSITION: She handed the presents to them.
Olivia made a cake for Emily, Sarah, and me.
Between whom did you sit?
TRANSITIVE VERB: Harry gave me the tickets.
Call me!
Did you take him to the movies?
In the second preposition example, two names appear between “for” and “me.” If this confuses you, eliminate
“Emily, Sarah, and” to get “Olivia made a cake for me.” Then you’ll see that “me” is the correct pronoun case, not “I”
(as in “Olivia made a cake for I”). This strategy of crossing out intervening words also works in spotting the correct
case for an object of a transitive verb.
In informal, spoken English, you will not hear “whom” used frequently, but in written English (particularly written
ACT English), you must remember the all important “m.” As in the third preposition example, “between whom” is
correct; “between who” is not. A good way to figure out if you should use “who” or “whom” in a sentence is to see
whether the sentence would use “he” or “him” (or “they” or “them”) if it were rearranged a little. If the sentence takes
“he” or “they,” you should use “who”; if it takes “him” or “them,” you should use “whom.”
If you rearrange “Between whom did you sit?” you get:
Did you sit between them?
Now you can see that you need to use “whom” in the original sentence.
The Possessive Case
You already know to use the possessive case when indicating possession of an object (see “The Possessive and
Pronouns” under “Apostrophes”):
My car
Her dress
Its tail
Whose wheelbarrow
You should also use the possessive case before a gerund, a verb form that usually ends with “ing” and is used as a
noun. For example,
When it comes to my studying for the ACT, “concentration” is my middle name.
Despite hours of practice, her playing is really terrible.
You can think of gerunds as turncoat verbs that are now nouns, so they need to be preceded by the same possessive
pronouns that precede noun objects.
The following chart shows you all the pronoun cases we’ve just discussed:
Nominative Case
Objective Case
Possessive Case
I
me
my
you (s.)
you
your
she
her
her
he
him
his
we
us
our
you (pl.)
you
your
they
them
their
it
it
its
who
whom
whose
Now that you know something about pronoun cases, try the following sample problem:
Me and Jesse went to Cosmic Bowling
4
4. F. NO CHANGE
G. Jesse and me
H.
Jesse and I
J.
I and Jesse
Night at the Bowladrome.
Knowing when to use “I” and when to use “me” can be difficult, especially within compound nouns. If you’re not sure
which is correct, use the crossing-out trick: cross out “and Jesse” and see what you have left.
Me went to Cosmic Bowling Night at the Bowladrome.
Unless you’re doing your Ralph Wiggum imitation, that sentence sounds (and is) wrong. The correct sentence?
Jesse and I went to Cosmic Bowling Night at the Bowladrome.
So the answer to the problem is H. Choice J, which also contains the correct pronoun “I,” is wrong because the
conventional rules of grammar require that you show a little deference in forming sentences involving yourself. “I”
should always come after the other people involved in the activity.
Verb Tenses
Most verb tense errors on the English Test will be pretty easy to spot, since we don’t often make tense errors in
everyday speech. When you read a tense error on the test, it will most likely sound wrong to you. Your ear is your
most reliable way of spotting tense errors.
Different Verb Tenses in One Sentence
Nowhere is it written that you must use the same tense throughout a sentence. For example, you can say, “I used to
eat chocolate bars exclusively, but after going through a conversion experience last year, I have broadened my range
and now eat gummy candy too.” That sentence has tense switches galore, but they are logical: the sentence uses past
tense when it talks about the past, and present tense when it talks about the present, and the progression from past
to present makes sense. Another acceptable example:
They are the best team in baseball, and I think they will triumph over what could could have been devastating injuries.
But you can’t throw in different tenses willy-nilly. They have to make sense. You can’t say:
Next year, I was on an ocean voyage.
“Next year” refers to the future, and “was” refers to the past. The sentence doesn’t make any sense unless you’re
doing some time travel. Your most powerful weapon against tense switch questions is logic. We could prattle on for
paragraph after paragraph about present tense, simple past, general present, and present perfect, but remembering
the millions of different tense forms, and when to use which, is both difficult and unnecessary. For the English Test,
if you don’t hear an error the first time you read a sentence, and you don’t see a pronoun problem, check out the
tenses and figure out whether they’re OK.
Tricky Verbs You’re Likely to See on the ACT
By tricky verbs, we mean those verbs that never sound quite right in any tense—like “to lie,” or “to swim.” When do
you lay and when do you lie? When do you swim and when have you swum? Unfortunately, there’s no easy memory
trick to help you remember when to use which verb form. The only solution is to learn and remember.
You LIE down for a nap.
You LAY something down on the table.
You LAY down yesterday.
You SWIM across the English Channel.
You SWAM across the Atlantic Ocean.
You HAD SWUM across the bathtub as a child.
“To lie” and “to swim” aren’t the only two difficult verbs. Below, you’ll find a table of difficult verbs in their infinitive,
simple past, and past participle forms. You don’t have to memorize all of these forms; you’ll probably only see one
tricky-verb question. Still, it is well worth your time to read the list below carefully, and especially to make sure you
understand those verbs that you’ve found confusing before.
Infinitive
Simple Past
Past Participle
Infinitive
Simple Past
Past Participle
Arise
Arose
Arisen
Lead
Led
Led
Become
Became
Become
Lie (to recline)
Lay
Lain
Begin
Began
Begun
Lie (tell fibs)
Lied
Lied
Blow
Blew
Blown
Put
Put
Put
Break
Broke
Broken
Ride
Rode
Ridden
Choose
Chose
Chosen
Ring
Rang
Rung
Come
Came
Come
Rise
Rose
Risen
Dive
Dived/dove
Dived
Run
Ran
Run
Do
Did
Done
See
Saw
Seen
Draw
Drew
Drawn
Set
Set
Set
Drink
Drank
Drunk
Shake
Shook
Shaken
Drive
Drove
Driven
Shine
Shone
Shone
Drown
Drowned
Drowned
Shrink
Shrank
Shrunk
Dwell
Dwelt/dwelled
Dwelt/dwelled
Shut
Shut
Shut
Eat
Ate
Eaten
Sing
Sang
Sung
Fall
Fell
Fallen
Sink
Sank
Sunk
Fight
Fought
Fought
Sit
Sat
Sat
Flee
Fled
Fled
Speak
Spoke
Spoken
Fling
Flung
Flung
Spring
Sprang
Sprung
Fly
Flew
Flown
Sting
Stung
Stung
Forget
Forgot
Forgotten
Strive
Strove/strived
Striven/strived
Freeze
Froze
Frozen
Swear
Swore
Sworn
Get
Got
Gotten
Swim
Swam
Swum
Give
Gave
Given
Swing
Swung
Swung
Go
Went
Gone
Take
Took
Taken
Grow
Grew
Grown
Tear
Tore
Torn
Hang (a thing)
Hung
Hung
Throw
Threw
Thrown
Hang (a person)
Hanged
Hanged
Wake
Woke
Woke/woken
Know
Knew
Known
Wear
Wore
Worn
Lay
Laid
Laid
Write
Wrote
Written
The ACT writers are going to get a little sneaky and use the tenses we do get wrong when we talk. One notoriously
annoying trick is the difference between “lie” and “lay” and all their variations. Here are the rules:
LIE: to recline or to disguise the truth
RIGHT: We lie down on the hammocks when we want to relax.
I lie to my mother about eating the cookies.
LAY: to place
RIGHT: Just lay down that air hockey table over there.
I lay the book on the table.
The tricky part is that the past tense of “lie” is “lay.”
She lay down yesterday, and today she’ll lie down again.
The past tense of “lay” is “laid.”
She laid down the law with an iron fist.
The Conditional
Another thorny tense issue arises with something called the conditional. The conditional is the verb form we use to
describe something uncertain, something that’s conditional upon something else. You can memorize the conditional
formula; it goes “If . . . were . . . would.” Look at this sentence:
WRONG: If I were running for president, my slogan will be “I’ll Fight for Your Right to Party.”
The use of “will be” in this sentence is wrong because you’re not certain you’re going to run for president (as
suggested by “If I were”); consequently, the word “will” is too strong. “Will” implies you’re definitely going to
campaign for president. You should use “would” instead—the conditional form of “will”—to indicate that running is
still only a possibility.
RIGHT: If I were running for president, my slogan would be “I’ll Fight for Your Right to Party.”
Notice also that the correct form is “If I were” not “If I was.” You’ll often hear people use “was” incorrectly in “If . . .”
phrases like this, but now you’ll know better. Sentences beginning with “If . . .” call for the subjunctive form of the
verb. In English, the subjunctive is often the same as the regular past tense verb, but in certain cases, notably to be,
the forms are irregular:
If I were, you were, s/he were, we were, they were, who were, it were
Adverbs and Adjectives
The ACT writers will test you once or twice on your ability to use adjectives and adverbs correctly in sentences. To
describe a noun, you use an adjective. To describe a verb, adjective, or adverb, you use an adverb. Look at the
following example:
WRONG: My mom made a well dinner.
RIGHT: My mom made a good dinner.
Since “dinner” is a noun, the descriptive word modifying it should be an adjective.
Now look at this example:
WRONG: My mom made dinner good.
RIGHT: My mom made dinner well.
Here, the word modified is “made,” a verb, so the descriptive word modifying it should be an adverb. Don’t let the
placement of the adverb fool you: just because it’s next to the noun “dinner” doesn’t mean that “dinner” is the word
modified. Often, though, you will find the modifier next to the modified word:
WRONG: I didn’t do good in the game last night.
RIGHT: I didn’t do well in the game last night.
In the example above, how the athlete did (a verb) is being described, so you need an adverb (“well”) rather than an
adjective (“good”).
Adverb/adjective errors are pretty common in everyday speech, so don’t rely entirely on your ear.
WRONG: She shut him up quick.
RIGHT: She shut him up quickly.
WRONG: I got an A easy.
RIGHT: I got an A easily.
The wrong examples above may sound familiar to you from everyday speech, but they are incorrect in written
English.
Idioms
You should trust your ear when you’re being tested on idioms. Idioms are expressions and phrasings that are
peculiar to a certain language—in the ACT’s case, the English language. They include odd expressions like “through
the grapevine” and “rain check” as well as simple ones like “bring up” (meaning “raise”). Idiom questions on the
English Test will often ask you to identify the correct prepositions used in certain expressions. This task is difficult
because there are no laws governing idioms. You have to be able to read a sentence and think, “That sounds plain old
wrong.” Fortunately, you probably won’t encounter more than a few idiom errors on the English Test you take. Take
a look at this idiom error:
WRONG: We spent days wading into the thousands of pages of rep orts.
“Wading into” sounds wrong. Instead, we say:
RIGHT: We spent days wading through the thousands of pages of reports.
Why do we use some prepositions instead of others? That’s just the way it is. The following is a list of proper
idiomatic usage:
He can’t abide by the no-spitting rule.
It’s terrible to discriminate against parakeets.
She accused me of stealing.
I have a plan to escape from this prison.
I agreed to eat the broccoli.
There’s no excuse for your behavior.
I apologized for losing the hamsters.
You can’t hide from your past.
She applied for a credit card.
It was all he’d hoped for.
She pretends to approve of my boyfriend.
I must insist upon it.
She argued with the bouncer.
It’s impossible to object to her arguments.
I arrived at work at noon.
I refuse to participate in this discussion.
You believe in ghosts.
Pray for me.
I can’t be blamed for your neuroses.
Protect me from evil.
Do you care about me?
Provide me with plenty of Skittles.
He’s in charge of grocery shopping.
She stayed home to recover from the flu.
Nothing compares to you.
I rely on myself.
What is there to complain about?
She stared at his chest.
He can always count on money from his mommy.
He subscribes to several trashy magazines.
Ice cream consists of milk, fat, and sugar.
I succeeded in fooling him.
I depend on no one.
Wait for me!
That’s where cats differ from dogs.
Work with me, people!
Comparative and Superlative Modifiers
Comparative modifiers compare one thing to another, while superlative modifiers tell you how one thing compares
to everything else. For example:
COMPARATIVE: My boyfriend is hotter than yours.
That purple-and-orange spotted dog is weirder than the blue cat.
Dan paints better than the other students.
SUPERLATIVE: My boyfriend is the hottest boy in the world.
That purple-and-orange spotted dog is the weirdest pet on the block.
Of all the students, Dan paints best.
You will probably see only one or two comparative and superlative modifier questions on the English Test, and they
will likely ask you to distinguish between the two types of modifiers. Remember that comparative modifiers are used
in relative statements; in other words, they compare one thing to another. Just because my boyfriend is hotter than
yours, it doesn’t mean that my boyfriend is hotter than Sue’s. However, if I used the superlative and told you that my
boyfriend is the hottest boy in the world, then there’s no way that Sue’s boyfriend is hotter than mine, unless, as is
probably the case, I’m exaggerating.
Comparative statements always require a comparison with something else. Simply saying “my boyfriend is hotter”
may get your meaning across in a heated dispute with your friends, but in proper English you need to finish that
sentence with a “than” phrase: “my boyfriend is hotter than Jude Law” or “my boyfriend is hotter than your dog.”
Sentence Structure
Sentence structure is the Big Deal when it comes to Usage/Mechanics problems. Of the 40 Usage/Mechanics
questions, almost half of them (18 to be exact) will test you on your knowledge of sentence structure, the topics of
which include:
1.
2.
3.
4.
5.
6.
7.
Connecting and Transitional Words
Subordinate or Dependent Clauses
Sentence Fragments
Comma Splices
Run-on Sentences
Misplaced Modifiers
Parallelism
Connecting and Transitional Words
We’ve already mentioned coordinating conjunctions (and, but, for, etc.) and transitional adverbs (however,
nevertheless, moreover, etc.) in “Punctuation.” Here you’ll learn more about these and other transitional words.
Coordinating Conjunctions
Coordinating conjunctions (and, but, or, nor, for, yet) connect words, phrases, and independent clauses of equal
importance in a sentence.
WORDS: You can hand the bottle to Seamus or Bea.
Liz and Amanda got down on the dance floor.
PHRASES: To get there, you must drive over a bridge and through a farm.
We walked by the park but not by the river.
CLAUSES: Tim can go to the store, or Jen can go instead.
It’s only ten o’clock, yet I feel really sleepy.
When joining two words or phrases, you should not use a comma, but (as demonstrated in “Commas”) if you have a
list of more than two words or phrases, commas should separate them and precede the conjunction. A comma also
needs to precede the coordinating conjunction when it joins two independent clauses, as in the sentence “Tim can go
to the store, or Jen can go instead,” above.
Transitional Adverbs
Like coordinating conjunctions, these adverbs (however, also, consequently, nevertheless, thus, moreover,
furthermore, etc.) can join independent clauses. When they do, they should be preceded by a semicolon (see
“Semicolons”) and followed, most of the time, by a comma. Short adverbs, such as “thus,” do not need a comma.
Here are some examples of transitional adverbs in action:
Joe always raves about soccer; however, he always refuses to watch a match.
If you can’t go to the prom with me, let me know as soon as possible ; otherwise, I’ll resent you and your inability to
communicate for the rest of my life.
You need to remember that transitional adverbs must be accompanied by semicolons. If you see a transitional
adverb on its own or preceded by a comma on the English Test, you should immediately know there’s an error.
Subordinating Conjunctions
When you have two independent clauses, but you feel that one is more important than the other, you can use a
subordinating conjunction to connect them. In other words, you use a subordinating conjunction (because, when,
since, after, until, although, before, etc.) to make one clause dependent on the other. By subordinating one clause,
you show the reader the relationship between the two clauses. For example, take the following two sentences:
I ate a rotten egg.
I became violently ill.
It seems likely that eating the rotten egg caused the violent illness. To make that relationship grammatically clear,
you can rephrase the sentences as:
Because I ate a rotten egg, I became violently ill.
Let’s try another example:
I found out my dog was really a rat.
I called the exterminator.
Put them through the subordinating conjunction transformation machine:
After I found out my dog was really a rat, I called the exterminator.
I called the exterminator after I found out my dog was really a rat.
In these examples, “I found out my dog was really a rat” becomes subordinate to “I called the exterminator.” You can
base your decision on which clause to subordinate by determining the relationship between the clauses. In the
example above, the discovery about the “dog” leads to the call; in other words, the discovery is the cause and calling
the exterminator the result. Subordinating the cause to the result often makes the most sense when forming these
sentences. For further discussion of this topic, move on to the next section.
Subordinate or Dependent Clauses
When you’re tested on subordinate conjunctions, you’ll need to select the most appropriate conjunction and place it
correctly within the sentence. When you’re tested on subordinate and dependent clauses, you’ll need to decide how
to form the whole sentence correctly. As touched upon above, not all clauses deserve the same emphasis in a
sentence. Equality is a good thing, but in the writing world you’ve got to give preference to some clauses over others.
You can run into problems if you’re too liberal with your coordinating conjunctions and transitional adverbs (the
adverbs that link independent clauses). These adverbs assume that the clauses being connected deserve equal weight
in a sentence. Take a look at this sentence:
Everyone regards Ginger as the most promising student in the class, and she gets the highest grades; also, she is the
president of the student council.
This sentence doesn’t read very well. Subordinating some of the clauses will improve the flow of the sentence:
Everyone regards Ginger as the most promising student in the class because she gets the highest grades and is the president
of the student council.
This new sentence explains why Ginger is “the most promising student” by subordinating the clauses that cite her
high grades and student council presidency.
Sentence Fragments
Sentence fragments are incomplete sentences that tend to look like this on the English Test:
We didn’t go outside. Even though the rain had stopped.
Tommy could not pay for his lunch. Having spent his last dollars on sunglasses.
Always a bit shy. She found herself unable to talk to the other kids.
The sentence fragments above are not sentences on their own. They can be attached to the independent clauses next
to them to form complete sentences:
We didn’t go outside, even though the rain had stopped.
Having spent his last dollars on sunglasses, Tommy could not pay for his lunch.
Always a bit shy, she found herself unable to talk to the other kids.
The answer choices on English Test questions will often make clear whether you should incorporate a fragment into
a neighboring sentence. For example:
We didn’t go outside. Even though the rain
17
17. A. NO CHANGE
B.
outside;
C. outside; even
D. outside, even
had stopped.
Notice how choices B, C, and D all give you the option of combining two sentences into one. That should give you a
good clue as to what’s required. The variation between the last three choices occurs in punctuation. If you agree that
A is incorrect, you can rely on your punctuation skills to decipher the correct answer. The answer, by the way, is D
because B and C, with their use of the semicolon, continue to isolate the sentence fragment from the sentence.
Other sentence fragment questions on the English Test will ask you to turn a fragment into its own full sentence
rather than simply to incorporate it into a different sentence. Again, you’ll be able to tell from the answer choices
what the ACT writers want:
We didn’t go outside. While the rain continued
18
18. F. NO CHANGE
G. Although the
H.
The
J.
Since the
to fall.
Answers F, G, and J don’t solve the sentence fragment problem. By choosing those, you still end up with a
subordinate clause posing as a sentence (G and J simply replace one subordinating conjunction with another). But
by getting rid of the subordinating conjunction altogether, you form a real sentence: “The rain continued to fall.” The
correct answer is H.
Most sentence fragments on the English Test will be subordinate or dependent clauses trying to be complete
sentences. By studying your subordinate and dependent clauses and learning what they look like, you’ll be able to
catch them committing sentence fragment crime.
Comma Splices
The ACT writers may test your ability to weed out illegal comma splices. A comma splice occurs when two
independent clauses are joined together by a comma with no intervening conjunction. For example,
Bowen walked to the park, Leah followed behind.
The comma between “park” and “Leah” forms a comma splice. Although the sentence may sound correct because the
comma demands a short pause between the two related clauses, the structure is wrong in written English. Instead,
two sentences are necessary:
Bowen walked to the park. Leah followed behind.
Or, if you explicitly want to show the relationship between the clauses, you can write:
Bowen walked to the park, while Leah followed behind.
OR
Bowen walked to the park, and Leah followed behind.
Inserting “while” subordinates the “Leah” clause to the “Bowen” clause. In the second sentence, the “and” joins the
two clauses on equal footing.
Think about the comma splice in construction terms: the comma (a wimpy nail) is too weak a punctuation mark to
join together two independent clauses (two big heavies). In order to join them, you have to add a conjunction (super
glue) to the comma or use a period (a bolt) instead.
Run-on Sentences
You can think of run-on sentences as comma splices minus the commas. For example:
Joan runs every day she is preparing for a marathon.
John likes to walk his dog through the park Kevin doesn’t.
To fix run-on sentences, you need to identify where they should be split. The first example should be broken into two
parts: “Joan runs every day” and “she is preparing for a marathon.” These are two independent clauses that can
stand on their own as sentences:
Joan runs every day. She is preparing for a marathon.
Alternatively, you may choose to show the relationship between these sentences by subordinating one to the other:
Joan runs every day because she is preparing for a marathon.
The second example, when split, becomes: “John likes to walk his dog through the park” and “Kevin doesn’t.” The
following sentences are correct alternatives to the original run-on:
John likes to walk his dog through the park . Kevin doesn’t.
John likes to walk his dog through the p ark, but Kevin doesn’t.
John likes to walk his dog through the park; however, Kevin doesn’t.
These are just a few ways you can join the two clauses. We could go on and on, showing different relationships
between the two clauses (but we won’t).
Misplaced Modifiers
Does the following sentence sound odd to you?
Having eaten six corn dogs, nausea overwhelmed Jane.
Nausea didn’t eat six corn dogs. Gluttonous Jane did. However, the sentence above says that nausea was the one
“having eaten six corn dogs.” This is a case of a misplaced modifier. When you have a modifier like “having eaten six
corn dogs,” it must come either directly before or directly after the word that it is modifying.
Having eaten six corn dogs, Jane was overwhelmed by nausea.
Jane, having eaten six corn dogs, was overwhelmed by nausea.
These two sentences make it clear that Jane was the one wolfing down the corn dogs.
Modifiers are not necessarily phrases like the one above. They can be adverbial phrases, adverbial clauses, or singleword adverb modifiers. You’ve already seen how adverbial-phrase modifiers work in the example above. The simple
rule for phrase modifiers is to make sure phrase modifiers are next to the word(s) they modify. The same rule
applies to clause modifiers. Misplaced clause modifiers look like this:
Bill packed his favorite clothes in his suitcase , which he planned to wear on vacation .
Now do you really think this guy is planning to wear his suitcase on vacation? Well, that’s what the sentence says.
It’ll be a pretty heavy outfit too, since the suitcase is packed with clothes. If Bill decides to wear his clothes instead of
his suitcase, you should say:
Bill packed his favorite clothes, which he planned to wear on vacation, in his suitcase.
Of course, he’ll be a slightly more conventional dresser, but the clothes will probably fit better than the suitcase.
The placement of single-word adverbs is slightly trickier than that of clause and phrase modifiers. You need to make
sure that adverb modifiers (such as just, almost, barely, even, and nearly) are modifying the word you intend them
to modify. If they aren’t, the sentence will probably still make sense, but it will have a different meaning than you
intended.
Take the sentence “Jay walked a half hour to the grocery store.” Now add to that sentence the adverbial modifier
“only.” The placement of “only” within the sentence will alter the meaning of the sentence:
Only Jay walked a half hour to the grocery store.
The sentence above means that no one but Jay made the walk.
Jay only walked a half hour to the grocery store.
Here, “only” modifies the verb “walked,” and the sentence means that Jay did nothing but walk—he didn’t run, and
he didn’t swim—to the store.
Jay walked only a half hour to the grocery store.
Hey, the walk to the grocery store isn’t too bad. According to the sentence above, it took Jay only a half hour to get
there.
Jay walked a half hour to only the grocery store.
Now we find out that Jay’s single destination was the grocery store (and we were about to accuse him of having
ulterior motives for taking that walk).
Parallelism
When you see a list underlined on the English Test, look for a parallelism error. Parallelism errors occur when items
in a list are mismatched. For example, if you have a list of verbs, then all items in the list must be verbs of the same
tense. For example,
WRONG: In the pool area, there is n o spitting, no running, and don’t throw your cigarette butts in the water.
The first two forbidden activities end in “ing” (they’re called gerunds, though that doesn’t really matter), and because
of that, the third activity must also end in “ing”.
RIGHT: In the pool area, there is no spitting, no running, and no throwing your cigarette butts in the water.
By simply converting the final verb to gerund form, you have parallel structure. Parallelism is also important when
you have expressions linked by the verb to be. Because you should think of to be as an equal sign, the words on either
side of the sign must be parallel. For example:
WRONG: To grow tired of London is growing tired of life.
RIGHT: To grow tired of London is to grow tired of life.
WRONG: Growing tired of London is to grow tired of life.
RIGHT: Growing tired of London is growing tired of life.
The examples above are not parallel when the verb forms are different on either side of “is.” You can make them
parallel by simply changing the form of one verb to the form of the other.
If you have a list of nouns, you must also maintain parallel construction. For example,
The personal ad said that she likes “ books, good food, and to take long walks on the beach.”
She apparently doesn’t like parallelism. “Books” and “food” are nouns, but “to take” is a verb infinitive. If she’s
hoping to get a call from the grammarian of her dreams, she should rewrite her ad to look like this:
The personal ad said that she likes “ books, good food, and long walks on the beach.”
Now that’s one grammatically correct lady.
Rhetorical Skills Questions on the English Test
THE ACT WRITERS BREAK RHETORICAL Skills questions into three categories:
1. Writing Strategy
2. Organization
3. Style
Some people may find these questions more challenging than the Usage/Mechanics questions because there are no
rules that strictly determine the Rhetorical Skills answers. Others may find them easier for that very reason—there’s
little to memorize. In any case, to answer Rhetorical Skills questions correctly you must develop an intuitive sense
for good English writing. We’ll show you how below.
Read the Whole Passage
Yes, we already gave you this advice in the “Strategies” section earlier in the book. But we think it’s such good advice
that we’ll give it again: you should read (or at least skim) the whole passage. You may want to underline key phrases
or transitions that help you decode the passage and that help you understand how its parts fit together. This strategy
is particularly important for answering Rhetorical Skills questions. Quite a few Rhetorical Skills questions demand
that you have a good understanding of the passage’s content, tone, and purpose. You won’t have that understanding
if you haven’t read (or at least skimmed) the entire passage.
Writing Strategy Questions
Writing strategy involves improving the effectiveness of a passage through careful revision and editing. Frequently,
strategy questions will ask you to choose the most appropriate topic or transitional sentence for a paragraph. Almost
as frequently, you will have to choose the best option for strengthening an argument by adding information or
evidence. In other questions, you may also have to choose which sections of an argument can be deleted. You will
also have to identify the purpose of a passage—its audience or its message—in other strategy questions.
The following strategy topics are covered in this section:
1. Transitions and Topic Sentences
2. Additional Detail and Evidence
3. Big Picture Purpose
Transitions and Topic Sentences
These questions ask you to figure out the best way to open or conclude paragraphs within a passage. Here’s an
example of a strategy question:
[2]
Victorian novelists were often
concerned with issues of character, plot,
and the Victorian social world. Dickens’s
novels, for example, were several-
hundred-page-long works documenting
the elaborate interweaving of his
characters.
[3]
Their “modernist” novels tended
The writer wishes to begin
47.
Paragraph 3 with a sentence
that strengthens the focus of
the paragraph, while providing
a transition from Paragraph 2.
Which of the following would
be the best choice?
A.
In the early twentieth
century, novelists began to
reject the Victorian emphasis
on social context and look for
a new focus for the novel.
B. Victorian novels ended with
the Victorian era.
C.
In the early twentieth
century, novelists further
developed this emphasis on
characters’ inner lives.
D.
World War I significantly
affected British culture in
the twentieth century.
to focus on the characters’ inner lives,
which they depicted through a stylistic
technique called “stream of
consciousness.” Several of the bestknown modernist novels were written
in this stream-of-consciousness
style.
Question 47 asks you to choose a sentence that will simultaneously serve as a topic sentence (“a sentence that
strengthens the focus of the paragraph”) for Paragraph 3 and as a transition sentence between the two paragraphs
(“while providing a transition from Paragraph 2”). In order to answer this question correctly, you need to
understand what the two paragraphs are saying. We suggest that you reread Paragraph 3 first. By developing a good
sense of what that paragraph says, you can eliminate answer choices that clearly do not work as topic sentences.
After you’ve eliminated any choices, make sure that you understand Paragraph 2. From the remaining choices, you
can identify the best transition sentence.
Done that? We hope that you immediately eliminated choices B and D from your list of possible topic sentences.
Choice B talks exclusively about the Victorian novel, making it an inappropriate topic sentence for a paragraph on
modernist novels. Choice D doesn’t talk specifically about novels at all. Its focus is World War I, which is not
mentioned elsewhere in the paragraph. So now you’ve narrowed the selection down to A and C. These sentences
have similar constructions, but they say radically different things: choice A claims that twentieth-century novelists
rejected Victorian ideas, while choice C claims that they embraced and developed Victorian ideas. In order to figure
out which one of these claims is true, you need to have read Paragraph 2 in addition to Paragraph 3. Paragraph 2
tells you that Victorian novelists were primarily concerned with the social world. In Paragraph 3, you discover that
modernist novelists were primarily concerned with characters’ thoughts and inner lives. Thus Paragraph 3 describes
a change in novel writing that occurred between the Victorian era and the early twentieth century. The correct
answer to the question is A.
The example above is fairly typical of transition and topic sentence questions you will encounter on the English Test.
Sometimes you’ll be asked to select only a topic sentence or only a transition sentence from the answer choices.
Those questions are usually less complex than the example above because you have to perform one fewer step. You
may also be asked to choose a concluding sentence for a paragraph. These questions are similar to transition
questions because a good concluding sentence tends to be one that easily and sensibly makes the transition to the
next paragraph.
Additional Detail and Evidence
These questions ask you to flesh out a paragraph by selecting the answer choice that provides the best additional
detail or evidence. For example,
[3]
Their “modernist” novels tended
to focus on the characters’ inner lives,
which they depicted through a stylistic
technique called “stream of
consciousness.” Several of the best-
known modernist novels were written
in this stream-of-consciousness
style.
The writer wishes to add
48.
information here that will
further support the point made
in the preceding sentence.
Which of the following
sentences will do that best?
F.
Today, this style is not as
popular as it once was.
G. However, there are many famous
early twentieth-century works
not written in this style.
H. Joyce’s Ulysses, for example,
was written in this style, and
it is widely considered one of
the most important books of
the century.
J.
Ford’s The Good Soldier,
although less read today, is a
great example of this style.
This question asks for additional information to support the point of the preceding sentence (“Several of the bestknown modernist novels were written in this stream-of-consciousness style”). To answer this question correctly, you
need to understand the point being made, so read the sentence carefully. You should be able to eliminate choices F
and G immediately. Choice F talks about the popularity of this style among contemporary authors—an issue that the
preceding sentence does not address. You can eliminate choice G almost immediately because it starts with
“however,” which indicates that it is going to make a statement that attempts to contradict, not support, the previous
point. Now you’ve successfully limited the answer choices to H and J. Both would provide the paragraph with an
example of a stream-of-consciousness work. The key to deciding which of these sentences is correct lies in the
preceding sentence, which talks about the “best-known modernist novels.” On the one hand, choice J tells you that
The Good Soldier is “less read today” and also, presumably, less well known. On the other hand, choice H tells you
that Ulysses is “widely considered one of the most important books of the century.” This statement suggests that the
novel is famous, so choice H is the best answer to the question.
Big Picture Purpose
On each English Test, you’ll probably encounter a few Big Picture Purpose questions. These questions always come
at the end of a passage. We call them Big Picture Purpose questions because they ask you to look at the big picture
and identify a passage’s main point, intended purpose, or intended audience.
These questions in many ways resemble some of the questions on the Reading Test. BPP questions do, after all, test
your comprehension of the passage—and comprehension is also what the Reading Test assesses. Because these
questions test your overall comprehension, they are difficult to prepare for outside the context of a whole passage.
Therefore, we suggest you prepare for these questions by studying our Reading Test chapter.
Before you start flipping through the book, we’ll give you an idea of how these questions look on the English Test.
They will often be phrased like this:
Suppose the writer has been assigned to write an essay explaining the development of the British novel from 1799 to 1945.
Would this essay successfully fulfill the assignment?
The answer choices to these questions come in two parts: the first part will respond either “No” or “Yes” to the
question, and the second part will give an explanation for this answer. For example,
A.
No, because the essay restricts its focus to the American novel from 1850 to 1945.
B.
No, because the essay omits mention of famous poets.
C.
Yes, because the essay focuses on the novel’s birth in the eighteenth century.
D. Yes, because the essay describes changes in novel writing from the end of the French Revolution to the end of World
War II.
Without reading the entire passage, you’re probably unable to answer a definite “No” or “Yes” to this question, but
you can eliminate an incorrect answer or two because of irrelevant or nonsensical explanations. In this example, you
can immediately cross off choice B because the explanation calls for a discussion of famous poets in the essay.
Famous poets, however, do not necessarily belong in an essay on the novel’s development. You can also cross off
choice C. It claims that the passage successfully fulfills the essay requirements because it discusses the novel’s birth
in the eighteenth century. However, the assignment calls for a discussion of the novel starting in 1799 (the end of the
eighteenth century), so choice C cannot be correct. By reading and understanding the passage, you’ll be able to
choose from the two remaining answers. If the passage indeed focuses on the American novel, choice A is correct,
and the essay does not succeed; if the essay describes the novel from the end of the French Revolution (1799) to the
end of the World War II (1945), choice D is correct, and the essay does succeed.
Organization
Organization questions deal with the logical structuring of the passage on the level of the sentence, the paragraph,
and the passage as a whole. These questions ask you to organize sections to maximize their coherence, order, and
unity by asking three types of questions:
1. Sentence Reorganization
2. Paragraph Reorganization
3. Passage Reorganization
Sentence Reorganization
Sentence reorganization questions often involve the placement of a modifier within a sentence. Your ability to
reorder a sentence correctly will depend on how well you have absorbed your grammar lessons above—specifically
the “Misplaced Modifiers” section. For example,
Austen wrote about a society of manners, in
which love triumphs over a rigid social hierarchy
despite confinement to her drawing room.
43. A.
43
NO CHANGE
B. (Place after love)
C. (Place after Austen)
D. (Place after society)
You probably guessed that the underlined phrase does not modify “hierarchy,” “love,” or “society.” The pronoun
“her” in the underlined phrase should tip you off that “Austen” is being modified. If you read “Misplaced Modifiers”
in the previous chapter, you should already know the cardinal rule of placing the modifier next to the modified word.
So the correct answer is C because the underlined part modifies “Austen.”
Approximately half of the organization questions on the English Test will ask you to reorder sentences. All of these
sentence reorganization problems will look similar to the one above. Study up on your modifier placement in order
to get them right.
Paragraph Reorganization
A couple of questions will ask you to reorder sentences within a paragraph. They will look much like this:
[1] In April, I’m usually in a bad mood because
of my debilitating pollen allergies. [2] In
November, despite the graying trees and the
short days, I’m elated because I can
celebrate both Thanksgiving and my birthday.
[3] My mood changes with the months.
[4] In the summer months I feel happy
from days spent in the sun.
61.
Which of the following
provides the most logical
ordering of the sentences in
the preceding paragraph?
A.
1, 4, 3, 2
B.
3, 4, 2, 1
C.
3, 1, 4, 2
D.
2, 1, 4, 3
The best way to approach these questions is to decide which sentence should come first, and then to eliminate
incompatible answer choices. Ask yourself: which sentence logically comes first in this sequence? Sentence 3 makes
a good topic sentence because it provides a general argument that can be followed and supported by examples. By
deciding that Sentence 3 should come first, you can immediately eliminate choices A and D because they do not
begin with Sentence 3. Now you can move on to arranging the rest of the paragraph. Each of the remaining
sentences talks about a different time of year: April, summer, and November. The three sentences should fall in that
chronological order (April, summer, November), as this is the most logical arrangement in this example. Therefore,
the correct answer is C.
If you are totally lost on a paragraph reorganization question, you can often look to the answer choices for clues. You
can look at the first sentences given to you by the answer choices and see whether any of them sound like topic
sentences. If you can identify a topic sentence, you’re well on your way to getting the correct answer.
Passage Reorganization
These appear at the end of passages. They will ask you either to insert a sentence where it best belongs in the passage
or to move a paragraph to a different location in the passage. Questions that ask you to insert a sentence will
generally look like this:
72. The writer wishes to include
the following sentence in the
essay: “That summer, I spent
so much time on the beach that
I could smell only a
combination of sand and
seaweed when I finally
returned to school.” That
sentence will fit most
smoothly and logically into
Paragraph:
F. 2, before the first sentence.
G. 3, after the last sentence.
H. 4, before the first sentence.
5, after the last sentence.
J.
This question is basically a strategy question disguised as an organization question. It asks you to identify the
sentence provided as an appropriate topic or concluding sentence for Paragraphs 2, 3, 4, or 5. When the answer
choice calls for the sentence to be placed “before the first sentence,” then it would become the topic sentence of the
paragraph. When the answer choice calls for the sentence to be placed “after the last sentence,” then it would
become the concluding sentence.
Questions that ask you to relocate a paragraph will generally look like this:
74. For the sake of the unity and
coherence of this essay,
Paragraph 4 should be placed:
F.
where it is now.
G.
after Paragraph 1.
H.
after Paragraph 2.
J.
after Paragraph 5.
To answer this question, look at (and perhaps underline) the topic sentences of each paragraph. These topic
sentences, removed from the passage, should follow a logical chain of thought. For example, look at these topic
sentences:
Topic Sentence
1:
Seasonal variations affect many aspects of my life.
Topic Sentence
2:
This April, the sight of leaves and the sounds of returning birds cheered me so much that I hugged a
tree.
Topic Sentence
3:
The return of the warm weather also meant that I got some much -needed exercise after being stuck
indoors all winter.
Topic Sentence
4:
My mood changes with the months.
Topic Sentence
5:
The weather’s effect on my mood and my fitness always reminds me of the undeniable connection
between people and nature.
Even without reading the whole passage, you can take an educated stab at the correct answer. Consider the logical
organization of an essay: introduction, supporting paragraphs, and conclusion. According to this structure, Topic
Sentence 1 should present the passage’s argument, and it should be followed by three paragraphs supporting the
argument and a final paragraph presenting a conclusion.
Now take a look at Topic Sentence 4. It makes a general argument about the weather’s effect on the author’s mood.
Ask yourself where the paragraph best fits into the passage: is it a supporting paragraph or a conclusion? It’s
unlikely that Paragraph 4 is a conclusion because it narrows the focus of the essay to talk about the author’s mood,
while other paragraphs in the essay discuss the author’s physical condition. If it’s a supporting paragraph, then
where does it belong? Eliminating choice J (which would make it the conclusion) leaves you with three options for a
supporting paragraph.
Your next step should be to take a look at the remaining Topic Sentences. Topic Sentence 2 also discusses the
weather’s effect on the author’s mood, but it deals specifically with April weather. Topic Sentence 3 discusses the
weather’s effects on the author’s physical health. If you choose Choice F and keep Paragraph 4 where it is, the
passage will be ordered like this: introduction, weather/mood, weather/health, weather/mood, conclusion. This
order doesn’t make much sense because it inexplicably divides the weather/mood discussions. Choices G and H
place the weather/mood paragraphs side by side. Choice G puts Paragraph 4 (general weather/mood) before
Paragraph 2 (April weather/mood), while choice H puts 2 before 4. When writing an essay, moving from the general
to the specific makes more sense than moving in the opposite direction because you want to support your claims
with specific evidence. So by using good writing strategy, you will arrive at the correct answer: G.
Style
Style questions generally concern effective word choice. They often ask you to choose the most appropriate word for
a sentence in terms of its tone and clarity. Other times, they’ll ask you to eliminate redundant words or phrases. In
the next section, we discuss the following style topics:
1. Redundancy
2. Appropriate Word Choice and Identifying Tone
Redundancy
The ACT writers will test you on your ability to spot redundant statements. Redundant statements say the same
thing twice, and you should always avoid redundancy on the English Test (in life too, if possible). For example,
WRONG: The diner closes at 3 a.m. in the morning.
RIGHT: The diner closes at 3 a.m.
“In the morning” is redundant because it is implied in “a.m.” Here’s another example of a redundant statement:
WRONG: In my opinion, I think we should go get some food.
“I think” and “In my opinion” mean the same thing, so you can eliminate one of the phrases from the sentence:
RIGHT: In my opinion, we should go get some food.
ALSO RIGHT: I think we should go get some food.
Either one of those phrases gets the point across; using both merely makes the sentence cumbersome.
Redundancy questions almost always give you the option to “OMIT the underlined portion.” If you spot a phrase or
word that means the same thing as the underlined portion, then you should always choose to “OMIT.” (For more on
“OMIT,” refer to “If the Phrase Doesn’t Fit, You Must ‘OMIT’” on p. .)
Appropriate Word Choice and Identifying Tone
Identifying the appropriate word choice can be as simple as figuring out whether a sentence should use the word
“their,” “there,” or “they’re.” But word choice can also be more complicated, involving many words working together
to create a tone. For example, the sentence “Lloyd George rocks!” probably does not belong in an essay on World
War I. It doesn’t fit because it’s written in a casual, slangy tone, and history essays are generally neither casual nor
slangy. The sentence might belong, however, in a passage on your awesome new friend, Lloyd George.
The content of a passage will generally give you a clue about the appropriate tone. Essays on history and culture will
probably be written in a fairly formal style—a style that omits youthful slang, casual contractions, and familiar
personal pronouns (such as “I” and “you”). A personal essay on your experiences driving a bulldozer, on your greatgrandmother, or on your new skateboard calls for a relatively informal style of writing. These personal essays can
exhibit varying degrees of informality. An essay by a young writer may be more colloquial and relaxed than an essay
by a mature writer recalling past experiences.
Tone is one of the most important elements in correctly answering word choice questions. You will encounter quite a
few questions that look like this:
During the Great War, the British Public believed
that Lloyd George rocks! He was widely admired
7
7. A.
NO CHANGE
B.
rocked!
C.
was an effective political
leader.
D. had the ability to unify the
government and thus to unify
Britain.
for his ability to unify the government and thus
to unify Britain.
Because we already told you that informality does not belong in a history essay, you can immediately eliminate
choices A and B, even though B correctly changes the verb tense. If you read the section above, you should also be
able to eliminate D because it is redundant—it repeats the information given in the next sentence. That leaves the
correct answer, C.
Practice Questions: Commas
1. For the Thanksgiving reunion, relatives were sitting in the dining room, on the porch, and in the carport.
A. Thanksgiving, reunion
B. Were, sitting
C. Porch and
D. No error
2. Lydia seems to be a kind, considerate girl.
A. Seems, to
B. Considerate, girl
C. Kind considerate
D. No error
3. This fishing pole Nathan, has seen better days.
A. Pole, Nathan,
B. Has, seen
C. Nathan,
D. No error
4. My cousin has moved to 56 Central Street Narragansett, Rhode Island 02882.
A. Has moved,
B. Central Street,
C. 56, Central
D. No error
5. The badger, a shy animal sometimes makes friends with a coyote.
A. Sometimes, makes
B. Friends, with
C. A shy animal,
D. No error
6. After the death of Blackbeard, the famous pirate, piracy disappeared from the coast of the American colonies.
A. The famous pirate
B. After the death,
C. Coast, of
D. No error
7. “Silent Night” was written by two men from the village of Oberndorf Austria.
A. men, from
B. Silent Night,
C. Oberndorf, Austria
D. No error
8. On November 19, 1929 Admiral Richard E. Byrd flew the Floyd Bennett to the base of the Queen Maud
Mountains.
A. Base, of
B. The, Queen
C. 1929,
D. no error
9. Oh I forgot to bring the cookies.
A. Oh,
B. I, forgot
C. To, bring
D. No error
10. “The boy in the kayak,” whispered Sue “is the new football captain."
A. Boy, in the
B. New, football
C. Whispered Sue,
D. No error
Answer Key
1. D
2. D
3. A
4. B
5. C
6. D
7. C
8. C
9. A
10. C
Practice Questions: Basic Grammar
1. Everyone in the bank-including the manager and the tellers, ran to the door when the fire alarm rang.
A. tellers, ran
B. tellers:ran
C. tellers, had run
D. tellers-ran
E. tellers' ran”
2. To no ones surprise, Joe didn't have his homework ready.
A. no ones surprise
B. noones surprise
C. no-ones surprise
D. no ones' surprise
E. no one's surprise
3. If he would have read “The White Birds,” he might have liked William Butler Yeats's poetry.
A. would have read
B. could have read
C. would of read
D. could of read
E. had read
4. After the hurricane, uprooted trees were laying all over the ground.
A. were laying
B. lying
C. were lying
D. were laid
E. was laid
5. Ralph Waldo Emerson (1803-1882), the great Transcendentalist philosopher, wrote in his essay “SelfReliance” of the need for an individual to develop his capacities.
A. essay “Self–Reliance”
B. essay, “Self-Reliance”
C. essay: Self-Reliance
D. essay, Self-Reliance
E. essay; “Self-Reliance”
6. The recently built children's amusement park has been called “ a boon to the community “ by its supporters
and “an eyesore” by its harshest critics.
A. and “an eyesore” by its harshest
B. and, “ an eyesore,” by its harshest
C. and, an eyesore; by its harshest
D. and-an eyesore- by its' harshest
E. and-“an eyesore”- by its' harshest
7. I always have trouble remembering the meaning of these two common verbs, affect (to change” or “to
influence”) and effect (“to cause” or “ to accomplish ) . “
A. “ to accomplish ). “
B. “ to accomplish” ).
C. “to accomplish).
D. To accomplish.
E. ( “ to accomplish. “ )
8. My class just finished reading- “ The Fall of the House of Usher “, a short story by Edgar Allen Poe.
A. reading- “ The Fall of the House of Usher”,
B. reading, The Fall of the House of Usher,
C. reading “The Fall of the House of Usher, “
D. reading, The Fall of the house of Usher, “
E. reading: The Fall of the House of Usher9. After it was repaired it ran perfect again.
A. ran perfect
B. ran perfectly
C. could run perfect
D. could of run perfect
E. would run perfectly
10. "Are there two e's in beetle," asked Margo?
A. there two e's in beetle," asked Margo?
B. their two e's in beetle?" asked Margo.
C. there two e's in beetle," asked Margo?
D. there two e's in beetle?" asked Margo.
E. there two e's in beetle, asked Margo?
11. The circus audience received a well-deserved round of applause for the perfectly timed acrobatic stunt.
A. audience received a well-deserved
B. audience gave a well deserved
C. audience did receive a well deserved
D. audience gave a well-deserved
E. audience did get a well-deserved
12. Looking directly at me, my Mother said, “ These are your options: the choice is yours.”
A. Mother said, “ These are your options: the choice is
B. Mother said- these are your options, the choice is
C. Mother had said, These are your options; the choice is
D. Mother had said, “These are your options; the choice is
E. Mother said, “These are your options; the choice is
13. Porcupine is from Latin porcus, “pig,” and spina, “spine.”
A. porcus, “pig,” and spina, “spine.”
B. Porcus-pig and spina, “spine.”
C. Porcus-pig, and Spina, “spine.”
D. Porcus-Pig-,Spina-spine.
E. Porcus, “pig,” and spina “spine”.
14. Seeing the dolphins, some sharks, a killer whale, and a Moray eel made the visit to the marine park
worthwhile.
A. a killer whale, and a Moray eel made the visit
B. a killer whale, and a moray eel made the visit
C. a killer whale and a moray eel makes the visit
D. a killer whale and a Moray eel makes the visit
E. a killer whale and a moray eel made the visit
15. Still, the fact that a planet exists outside our solar system encourages hope that other solar systems exist, and
in them, perhaps, a planet that does support life.
A. that a planet exists outside our solar system encourages hope that other solar systems exist, and
B. that a Planet exists out side our solar system encourages hope that other solar systems exist and
C. could be that a planet exists outside our solar system encourages hope that other solar systems exist, and
D. that a planet exist outside our solar systems encourage hope that other solar systems exist, and
E. that a planet does exists out side our solar system encourages hope that other solar systems exist, and
16. Mail-order shopping can be convenient and timesaving with appropriate precautions, it is safe as well.
A. can be convenient and timesaving
B. can be convenient and timesaving;
C. should be convenient and time saving;
D. could be convenient and time saving;
E. can be convenient and time-saving;
17. Among the many fields of science, no matter what turns you on, there are several fields of study.
A. science, no matter what turns you on,
B. Science, no matter what turns you on,
C. Science, no matter which you chose,
D. Science, no matter which of these you choseE. science, no matter which you choose,
18. The fact that boxing is known to cause head injuries and brain damage should lead us to inform the public
and push for a ban on boxing.
A. should lead us to inform
B. could lead us to inform
C. should of led us to inform
D. will lead us to inform
E. should have led us to inform,
19. The first part of the test was on chemistry, the second on mathematics, and the third on english.
A. on mathematics, and the third on english.
B. on mathematics; and the third on English.
C. on Mathematics; and the third on English.
D. on mathematics, and the third on English.
E. on mathematics: and the third on English.
20. The Diary of Anne Frank showed a young girl's courage during two years of hiding.
A. showed a young girl's courage
B. shows a young girl's courage
C. did show a young girls courage
D. has shown a young girl's courage
E. showed a young girl's courage
21. In August my parents will be married for twenty-five years.
A. will be married for twenty-five years.
B. shall have been married for twenty-five years.
C. will have been married for twenty-five years.
D. will be married for twenty five years.
E. will have married for twenty-five years.
Answer Key
1. D
2. E
3. E
4. C
5. A
6. A
7. B
8. C
9. B
10. D
11. D
12. E
13. A
14. B
15. A
16. E
17. E
18. A
19. D
20. B
21. C
Practice Questions: Intermediate Grammar
1. The word boycott derives from the name of Charles C. Boycott, an English land agent in Ireland that was
ostracized for refusing to reduce rent.
A. that was ostracized for refusing
B. who was ostracized for refusing
C. which was ostracized for refusing
D. that had been ostracized for refusing
E. who had been ostracized for refusing
2. As a result of his method for early music education, Shinichi Suzuki has been known as one of the world's
great violin teachers.
A. has been known as one
B. had been known as one
C. is seen as one
D. is being seen as one
E. has been one
3. Last night the weather forecaster announced that this is the most rainy season the area has had in the past
decade.
A. this is the most rainy season the
B. this has been the most rainy season the
C. this was the most rainy season the
D. this is noted as the most rainy season the
E. this is the rainiest season the
4. Although Mandy is younger than her sister, Mandy is the tallest of the two.
A. is the tallest of the
B. is the taller of the
C. has been the taller of the
D. is the most tall of the
E. is the more taller of the
5. When Katherine Hepburn's play came to town, all the tickets had sold out far in advance.
A. had sold out far
B. have sold out far
C. were sold out far
D. had been sold out far
E. had been sold out for
6. The origins of most sports is unknown.
A. sports is unknown
B. sports have been unknown
C. sports are unknown
D. sports has been unknown
E. sports are now unknown
7. Neither of the Smith brothers expect to be drafted by a major league team this year.
A. expect to be drafted
B. expects to be drafted
C. has expected to be drafted
D. is expecting to be drafted
E. was expecting to be drafted
8. Has any of the witnesses been sworn in yet?
A. Has any of the
B. Is any of the
C. Will any of the
D. Are any of the
E. Have any of the
9. The Lusitania sunk on May 7, 1915.
A. sunk
B. did sink
C. was sunk
D. did sank
E. sank
10. Whos in the office now?
A. Whos in
B. Whose in
C. Who is in
D. Who's in
E. Whose' in
11. There are now many kinds of dictionaries, such as a dictionary of synonyms and antonyms, a biographical
dictionary, and a geographical dictionary with pronunciations given.
A. with pronunciations given
B. that has pronunciations given
C. with pronunciations' given
D. that have pronunciations given
E. that do have pronunciations given
12. Towering seven hundred feet above the valley floor, Mount Rushmore National Memorial was an
impressive site.
A. was an impressive site
B. is a impressive sight
C. is an impressive sight
D. was an impressive sight
E. is an impressive site
13. San Francisco lays southwest of Sacramento.
A. lays southwest
B. has laid southwest
C. is lying southwest
D. lain southwest
E. lies southwest
14. Did they know that Labor Day always came on the first Monday in September?
A. came on
B. comes on
C. has come on
D. had come on
E. has came on
15. Eating, drinking, and to stay up late at night were among her pleasures.
A. to stay up late
B. to remain up late
C. staying up late
D. she liked staying up late
E. trying to stay up late
16. Each night when night came and the temperature fell, my parents lit the fire in the bedroom.
A. and the temperature fell,
B. and that the temperature did fall
C. and that the temperature fell
D. and because the temperature fell
E. and when the temperature fell
17. Francis promised to bring the Papago basket that she bought in Arizona.
A. bought in
B. had bought in
C. has bought in
D. did buy in
E. purchased in
18. He has lain his racquetball glove on the beach.
A. has lain
B. has laid
C. have lain
D. have laid
E. is lying
19. I would have lent you my notes if you would have asked me.
A. would have asked me
B. could of asked
C. could ask
D. had asked
E. had of asked
20. Many scientists are still hoping to have found life on another planet.
A. to have found
B. to find
C. two find
D. to have been found
E. too have found
21. Because she had an astounding memory, Sue has never forgotten an important equation.
A. had an
B. could have had
C. has
D. did have
E. has had
Answer Key
1. B
2. C
3. E
4. B
5. D
6. C
7. B
8. E
9. E
10. D
11. A
12. C
13. E
14. B
15. C
16. E
17. B
18. B
19. D
20. B
21. C
Practice Questions: Advanced Grammar
1. David was known for belching; and telling inappropriate jokes in public.
A. Capitalization
B. Punctuation
C. Spelling
D. Grammar
2. Graduation from High School is considered by many a momentous occasion.
A. Capitalization
B. Punctuation
C. Spelling
D. Grammar
3. Nurses plays a vital role in the healthcare profession.
A. Capitalization
B. Punctuation
C. Spelling
D. Grammar
4. After having his tonsels removed, the child was listless for a few days.
A. Capitalization
B. Punctuation
C. Spelling
D. Grammar
5. The park was serine at twilight.
A. Capitalization
B. Punctuation
C. Spelling
D. Grammar
6. Was the patient's mind lucid during the evaluation.
A. Capitalization
B. Punctuation
C. Spelling
D. Grammar
7. The bachalor never married. Most people thought it was because of misogyny.
A. Capitalization
B. Punctuation
C. Spelling
D. Grammar
8. The intricacy of the mathematical equation, drove the student crazy trying to solve it.
A. Capitalization
B. Punctuation
C. Spelling
D. Grammar
9. The hybrid tomatoes is immune to most common diseases.
A. Capitalization
B. Punctuation
C. Spelling
D. Grammar
10. The professor was humiliated when his students reported him to the Dean for verbal abuse.
A. Capitalization
B. Punctuation
C. Spelling
D. Grammar
11. The con artist hoodwinked the old lady when he sold her fradulent insurance.
A. Capitalization
B. Punctuation
C. Spelling
D. Grammar
12. The movie star was accused of a misdemeanor, when she stole 15 dollars worth of merchandise from the
store.
A. Capitalization
B. Punctuation
C. Spelling
D. Grammar
13. The congregation sang a comtemporary hymn.
A. Capitalization
B. Punctuation
C. Spelling
D. Grammar
14. The wound were necrotic when examined.
A. Capitalization
B. Punctuation
C. Spelling
D. Grammar
15. The defendint exhibited a peevish appearance.
A. Capitalization
B. Punctuation
C. Spelling
D. Grammar
16. The band director was scheduled to play the piccolo on tuesday.
A. Capitalization
B. Punctuation
C. Spelling
D. Grammar
17. The renter was remiss; about the rent.
A. Capitalization
B. Punctuation
C. Spelling
D. Grammar
18. The old man was know for sapient knowledge.
A. Capitalization
B. Punctuation
C. Spelling
D. Grammar
19. The inventor create several specious ideas to solve the problem.
A. Capitalization
B. Punctuation
C. Spelling
D. Grammar
20. The teacher identified the troublemakers, in her classroom.
A. Capitalization
B. Punctuation
C. Spelling
D. Grammar
Answer Key
1. B
2. A
3. D
4. C
5. C
6. B
7. C
8. B
9. D
10. A
11. C
12. B
13. C
14. D
15. C
16. A
17. B
18. D
19. D
20. B
Practice Questions: Basic reading comprehension
1. Questions 1-7.
In the sixteenth century, an age of great marine and terrestrial exploration, Ferdinand Magellan led the first
expedition to sail around the world. As a young Portuguese noble, he served the king of Portugal, but he became
involved in the quagmire of political intrigue at court and lost the king's favor. After he was dismissed from
service to the king of Portugal, he offered to serve the future Emperor Charles V of Spain.
A papal decree of 1493 had assigned all land in the New World west of 50 degrees W longitude to Spain and all
the land east of that line to Portugal. Magellan offered to prove that the East Indies fell under Spanish authority.
On September 20, 1519, Magellan set sail from Spain with five ships. More than a year later, one of these ships
was exploring the topography of South America in search of a water route across the continent. This ship sank,
but the remaining four ships searched along the southern peninsula of South America. Finally they found the
passage they sought near a latitude of 50 degrees S. Magellan named this passage the Strait of All Saints, but
today we know it as the Strait of Magellan.
One ship deserted while in this passage and returned to Spain, so fewer sailors were privileged to gaze at that
first panorama of the Pacific Ocean. Those who remained crossed the meridian we now call the International
Date Line in the early spring of 1521 after ninety-eight days on the Pacific Ocean. During those long days at
sea, many of Magellan's men died of starvation and disease.
Later Magellan became involved in an insular conflict in the Philippines and was killed in a tribal battle. Only
one ship and seventeen sailors under the command of the Basque navigator Elcano survived to complete the
westward journey to Spain and thus prove once and for all that the world is round, with no precipice at the edge.
The sixteenth century was an age of great ___exploration.
A. cosmic
B. land
C. mental
D. common man
E. none of the above
2. Magellan lost the favor of the king of Portugal when he became involved in a political ___.
A. entanglement
B. discussion
C. negotiation
D. problems
E. none of the above
3. The Pope divided New World lands between Spain and Portugal according to their location on one side or the
other of an imaginary geographical line 50 degrees west of Greenwich that extends in a ___ direction.
A. north and south
B. crosswise
C. easterly
D. south east
E. north and west
4. One of Magellan's ships explored the ___ of South America for a passage across the continent.
A. coastline
B. mountain range
C. physical features
D. islands
E. none of the above
5. Four of the ships sought a passage along a southern ___.
A. coast
B. inland
C. body of land with water on three sides
D. border
E. answer not available
6. The passage was found near 50 degrees S of ___.
A. Greenwich
B. The equator
C. Spain
D. Portugal
E. Madrid
7. In the spring of 1521, the ships crossed the ___ now called the International Date Line.
A. imaginary circle passing through the poles
B. Imaginary line parallel to the equator
C. area
D. land mass
E. answer not found in article
8. Questions 8-14
Marie Curie was one of the most accomplished scientists in history. Together with her husband, Pierre, she
discovered radium, an element widely used for treating cancer, and studied uranium and other radioactive
substances. Pierre and Marie's amicable collaboration later helped to unlock the secrets of the atom.
Marie was born in 1867 in Warsaw, Poland, where her father was a professor of physics. At the early age, she
displayed a brilliant mind and a blithe personality. Her great exuberance for learning prompted her to continue
with her studies after high school. She became disgruntled, however, when she learned that the university in
Warsaw was closed to women. Determined to receive a higher education, she defiantly left Poland and in 1891
entered the Sorbonne, a French university, where she earned her master's degree and doctorate in physics.
Marie was fortunate to have studied at the Sorbonne with some of the greatest scientists of her day, one of
whom was Pierre Curie. Marie and Pierre were married in 1895 and spent many productive years working
together in the physics laboratory. A short time after they discovered radium, Pierre was killed by a horsedrawn wagon in 1906. Marie was stunned by this horrible misfortune and endured heartbreaking anguish.
Despondently she recalled their close relationship and the joy that they had shared in scientific research. The
fact that she had two young daughters to raise by herself greatly increased her distress.
Curie's feeling of desolation finally began to fade when she was asked to succeed her husband as a physics
professor at the Sorbonne. She was the first woman to be given a professorship at the world-famous university.
In 1911 she received the Nobel Prize in chemistry for isolating radium. Although Marie Curie eventually
suffered a fatal illness from her long exposure to radium, she never became disillusioned about her work.
Regardless of the consequences, she had dedicated herself to science and to revealing the mysteries of the
physical world.
The Curies' ____ collaboration helped to unlock the secrets of the atom.
A. friendly
B. competitive
C. courteous
D. industrious
E. chemistry
9. Marie had a bright mind and a __personality.
A. strong
B. lighthearted
C. humorous
D. strange
E. envious
10. When she learned that she could not attend the university in Warsaw, she felt___.
A. hopeless
B. annoyed
C. depressed
D. worried
E. none of the above
11. Marie ___ by leaving Poland and traveling to France to enter the Sorbonne.
A. challenged authority
B. showed intelligence
C. behaved
D. was distressed
E. answer not available in article
12. _____she remembered their joy together.
A. Dejectedly
B. Worried
C. Tearfully
D. Happily
E. Sorrowfully
13. Her ____ began to fade when she returned to the Sorbonne to succeed her husband.
A. misfortune
B. anger
C. wretchedness
D. disappointment
E. ambition
14. Even though she became fatally ill from working with radium, Marie Curie was never ____.
A. troubled
B. worried
C. disappointed
D. sorrowful
E. disturbed
15. Questions 15-19.
Mount Vesuvius, a volcano located between the ancient Italian cities of Pompeii and Herculaneum, has received
much attention because of its frequent and destructive eruptions. The most famous of these eruptions occurred
in A. D. 79.
The volcano had been inactive for centuries. There was little warning of the coming eruption, although one
account unearthed by archaeologists says that a hard rain and a strong wind had disturbed the celestial calm
during the preceding night. Early the next morning, the volcano poured a huge river of molten rock down upon
Herculaneum, completely burying the city and filling in the harbor with coagulated lava.
Meanwhile, on the other side of the mountain, cinders, stone and ash rained down on Pompeii. Sparks from the
burning ash ignited the combustible rooftops quickly. Large portions of the city were destroyed in the
conflagration. Fire, however, was not the only cause of destruction. Poisonous sulphuric gases saturated the air.
These heavy gases were not buoyant in the atmosphere and therefore sank toward the earth and suffocated
people.
Over the years, excavations of Pompeii and Herculaneum have revealed a great deal about the behavior of the
volcano. By analyzing data, much as a zoologist dissects a specimen animal, scientist have concluded that the
eruption changed large portions of the area's geography. For instance, it turned the Sarno River from its course
and raised the level of the beach along the Bay of Naples. Meteorologists studying these events have also
concluded that Vesuvius caused a huge tidal wave that affected the world's climate.
In addition to making these investigations, archaeologists have been able to study the skeletons of victims by
using distilled water to wash away the volcanic ash. By strengthening the brittle bones with acrylic paint,
scientists have been able to examine the skeletons and draw conclusions about the diet and habits of the
residents. Finally, the excavations at both Pompeii and Herculaneum have yielded many examples of classical
art, such as jewelry made of bronze, which is an alloy of copper and tin.
The eruption of Mount Vesuvius and its tragic consequences have provided us with a wealth of data about the
effects that volcanoes can have on the surrounding area. Today volcanologists can locate and predict eruptions,
saving lives and preventing the destruction of cities and cultures.
Herculaneum and its harbor were buried under ___lava.
A. liquid
B. solid
C. flowing
D. gas
E. answer not available
16. The poisonous gases were not ___ in the air.
A. able to float
B. visible
C. able to evaporate
D. invisible
E. able to condense
17. Scientists analyzed data about Vesuvius in the same way that a zoologist ___ a specimen.
A. describes in detail
B. studies by cutting apart
C. photographs
D. chart
E. answer not available
18. ____have concluded that the volcanic eruption caused a tidal wave.
A. Scientist who study oceans
B. Scientist who study atmospheric conditions
C. Scientist who study ash
D. Scientist who study animal behavior
E. Answer not available in article
19. Scientist have used ___water to wash away volcanic ash from the skeletons of victims.
A. bottled
B. volcanic
C. purified
D. sea
E. fountain
20. Questions 20-24.
Conflict had existed between Spain and England since the 1570s. England wanted a share of the wealth that
Spain had been taking from the lands it had claimed in the Americas.
Elizabeth I, Queen of England, encouraged her staunch admiral of the navy, Sir Francis Drake, to raid Spanish
ships and towns. Though these raids were on a small scale, Drake achieved dramatic success, adding gold and
silver to England's treasury and diminishing Spain's omnipotence.
Religious differences also caused conflict between the two countries. Whereas Spain was Roman Catholic, most
of England had become Protestant. King Philip II of Spain wanted to claim the throne and make England a
Catholic country again. To satisfy his ambition and also to retaliate against England's theft of his gold and
silver, King Philip began to build his fleet of warships, the Armada, in January 1586.
Philip intended his fleet to be indestructible. In addition to building new warships, he marshaled one hundred
and thirty sailing vessels of all types and recruited more than nineteen thousand robust soldiers and eight
thousand sailors. Although some of his ships lacked guns and others lacked ammunition, Philip was convinced
that his Armada could withstand any battle with England.
The martial Armada set sail from Lisbon, Portugal, on May 9,1588, but bad weather forced it back to port. The
voyage resumed on July 22 after the weather became more stable.
The Spanish fleet met the smaller, faster, and more maneuverable English ships in battle off the coast of
Plymouth, England, first on July 31 and again on August 2. The two battles left Spain vulnerable, having lost
several ships and with its ammunition depleted. On August 7, while the Armada lay at anchor on the French
side of the Strait of Dover, England sent eight burning ships into the midst of the Spanish fleet to set it on fire.
Blocked on one side, the Spanish ships could only drift away, their crews in panic and disorder. Before the
Armada could regroup, the English attacked again on August 8.
Although the Spaniards made a valiant effort to fight back, the fleet suffered extensive damage. During the
eight hours of battle, the Armada drifted perilously close to the rocky coastline. At the moment when it seemed
that the Spanish ships would be driven onto the English shore, the wind shifted, and the Armada drifted out into
the North Sea. The Spaniards recognized the superiority of the English fleet and returned home, defeated.
Sir Francis Drake added wealth to the treasury and diminished Spain's ____.
A. unlimited power
B. unrestricted growth
C. territory
D. treaties
E. answer not available in article
21. Philip recruited many ___soldiers and sailors.
A. warlike
B. strong
C. accomplished
D. timid
E. non experienced
22. The ____ Armada set sail on May 9, 1588.
A. complete
B. warlike
C. independent
D. isolated
E. answer not available
23. The two battles left the Spanish fleet ____.
A. open to change
B. triumphant
C. open to attack
D. defeated
E. discouraged
24. The Armada was ___ on one side.
A. closed off
B. damaged
C. alone
D. circled
E. answer not available in this article
25. Questions 25-29.
The victory of the small Greek democracy of Athens over the mighty Persian empire in 490 B. C. is one of the
most famous events in history. Darius, king of the Persian empire, was furious because Athens had interceded
for the other Greek city-states in revolt against Persian domination. In anger the king sent an enormous army to
defeat Athens. He thought it would take drastic steps to pacify the rebellious part of the empire. Persia was
ruled by one man.
In Athens, however, all citizens helped to rule. Ennobled by this participation, Athenians were prepared to die
for their city-state. Perhaps this was the secret of the remarkable victory at Marathon, which freed them from
Persian rule. On their way to Marathon, the Persians tried to fool some Greek city-states by claiming to have
come in peace. The frightened citizens of Delos refused to believe this. Not wanting to abet the conquest of
Greece, they fled from their city and did not return until the Persians had left. They were wise, for the Persians
next conquered the city of Etria and captured its people.
Tiny Athens stood alone against Persia. The Athenian people went to their sanctuaries. There they prayed for
deliverance. They asked their gods to expedite their victory. The Athenians refurbished their weapons and
moved to the plain of Marathon, where their little band would meet the Persians. At the last moment, soldiers
from Plataea reinforced the Athenian troops.
The Athenian army attacked, and Greek citizens fought bravely. The power of the mighty Persians was offset
by the love that the Athenians had for their city. Athenians defeated the Persians in archery and hand combat.
Greek soldiers seized Persian ships and burned them, and the Persians fled in terror. Herodotus, a famous
historian, reports that 6400 Persians died, compared with only 192 Athenians.
Athens had ____the other Greek city-states against the Persians.
A. refused help to
B. intervened on behalf of
C. wanted to fight
D. given orders for all to fight
E. defeated
26. Darius took drastic steps to ___ the rebellious Athenians.
A. weaken
B. destroy
C. calm
D. placate
E. answer not available
27. Their participation___to the Athenians.
A. gave comfort
B. gave honor
C. gave strength
D. gave fear
E. gave hope
28. The people of Delos did not want to ___ the conquest of Greece.
A. end
B. encourage
C. think about
D. daydream about
E. answer not available
29. The Athenians were ___by some soldiers who arrived from Plataea.
A. welcomed
B. strengthened
C. held
D. captured
E. answer not available
30. Questions 30-32.
The Trojan War is one of the most famous wars in history. It is well known for the ten-year duration, for the
heroism of a number of legendary characters, and for the Trojan horse. What may not be familiar, however, is
the story of how the war began.
According to Greek myth, the strife between the Trojans and the Greeks started at the wedding of Peleus, King
of Thessaly, and Thetis, a sea nymph. All of the gods and goddesses had been invited to the wedding
celebration in Troy except Eris, goddesses of discord. She had been omitted from the guest list because her
presence always embroiled mortals and immortals alike in conflict.
To take revenge on those who had slighted her, Eris decided to cause a skirmish. Into the middle of the banquet
hall, she threw a golden apple marked “for the most beautiful.” All of the goddesses began to haggle over who
should possess it. The gods and goddesses reached a stalemate when the choice was narrowed to Hera, Athena,
and Aphrodite. Someone was needed to settle the controversy by picking a winner. The job eventually fell to
Paris, son of King Priam of Troy, who was said to be a good judge of beauty.
Paris did not have an easy job. Each goddess, eager to win the golden apple, tried aggressively to bribe him.
“I'll grant you vast kingdoms to rule, “ promised Hera. “Vast kingdoms are nothing in comparison with my
gift,” contradicted Athena. “Choose me and I'll see that you win victory and fame in war.” Aphrodite outdid her
adversaries, however. She won the golden apple by offering Helen, Zeus' daughter and the most beautiful
mortal, to Paris. Paris, anxious to claim Helen, set off for Sparta in Greece.
Although Paris learned that Helen was married, he accepted the hospitality of her husband, King Menelasu of
Sparta, anyway. Therefore, Menelaus was outraged for a number of reasons when Paris departed, taking Helen
and much of the king's wealth back to Troy. Menelaus collected his loyal forces and set sail for Troy to begin
the war to reclaim Helen.
Eris was known for ___both mortals and immortals.
A. scheming against
B. involving in conflict
C. feeling hostile toward
D. ignoring
E. comforting
31. Each goddess tried ___to bribe Paris.
A. boldly
B. effectively
C. secretly
D. carefully
E. answer not stated
32. Athena ___ Hera, promising Paris victory and fame in war.
A. denied the statement of
B. defeated
C. agreed with
D. restated the statement
E. questioned the statement
33. Questions 33-37.
One of the most intriguing stories of the Russian Revolution concerns the identity of Anastasia, the youngest
daughter of Czar Nicholas II. During his reign over Russia, the Czar had planned to revoke many of the harsh
laws established by previous czars. Some workers and peasants, however, clamored for more rapid social
reform. In 1918 a group of these people, known as Bolsheviks, overthrew the government. On July 17 or 18,
they murdered the Czar and what was thought to be his entire family.
Although witnesses vouched that all the members of the Czar's family had been executed, there were rumors
suggesting that Anastasia had survived. Over the years, a number of women claimed to be Grand Duchess
Anastasia. Perhaps the best –known claimant was Anastasia Tschaikovsky, who was also known as Anna
Anderson.
In 1920, eighteen months after the Czar's execution, this terrified young woman was rescued from drowning in
a Berlin river. She spent two years in a hospital, where she attempted to reclaim her health and shattered mind.
The doctors and nurses thought that she resembled Anastasia and questioned heer about her background. She
disclaimed any connection with the Czar's family.
Eight years later, though, she claimed that she was Anastasia. She said that she had been rescued by two
Russian soldiers after the Czar and the rest of her family had been killed. Two brothers named Tschaikovsky
had carried her into Romania. She had married one of the brothers, who had taken her to Berlin and left her
there, penniless and without a vocation. Unable to invoke the aid of her mother's family in Germany, she had
tried to drown herself.
During the next few years, scores of the Czar's relatives, ex-servants, and acquaintances interviewed her. Many
of these people said that her looks and mannerisms were evocative of the Anastasia that they had known. Her
grandmother and other relatives denied that she was the real Anastasia, however.
Tried of being accused of fraud, Anastasia immigrated to the United States in 1928 and took the name Anna
Anderson. She still wished to prove that she was Anastasia, though, and returned to Germany in 1933 to bring
suit against her mother's family. There she declaimed to the court, asserting that she was indeed Anastasia and
deserved her inheritance.
In 1957, the court decided that it could neither confirm nor deny Anastasia's identity. Although we will
probably never know whether this woman was the Grand Duchess Anastasia, her search to establish her identity
has been the subject of numerous books, plays, and movies.
Some Russian peasants and workers___for social reform.
A. longed
B. cried out
C. begged
D. hoped
E. thought much
34. Witnesses ___ that all members of the Czar's family had been executed.
A. gave assurance
B. thought
C. hoped
D. convinced some
E. answer not stated
35. Tschaikovsky ____any connection with the Czar's family.
A. denied
B. stopped
C. noted
D. justified
E. answer not stated
36. She was unable to ___the aid of her relative.
A. locate
B. speak about
C. call upon
D. identify
E. know
37. In court she ___ maintaining that she was Anastasia and deserved her inheritance.
A. finally appeared
B. spoke forcefully
C. testified
D. gave evidence
E. answer not stated
38. Questions 38-39.
King Louis XVI and Queen Marie Antoinette ruled France from 1774 to 1789, a time when the country was
fighting bankruptcy. The royal couple did not let France's insecure financial situation limit their immoderate
spending, however. Even though the minister of finance repeatedly warned the king and queen against wasting
money, they continued to spend great fortunes on their personal pleasure. This lavish spending greatly enraged
the people of France. They felt that the royal couple bought its luxurious lifestyle at the poor people's expense.
Marie Antoinette, the beautiful but exceedingly impractical queen, seemed uncaring about her subjects; misery.
While French citizens begged for lower taxes, the queen embellished her palace with extravagant works of art.
She also surrounded herself with artists, writers, and musicians, who encouraged the queen to spend money
even more profusely.
While the queen's favorites glutted themselves on huge feasts at the royal table, many people in France were
starving. The French government taxed the citizens outrageously. These high taxes paid for the entertainments
the queen and her court so enjoyed. When the minister of finance tried to stop these royal spendthrifts, the
queen replaced him. The intense hatred that the people felt for Louis XVI and Marie Antoinette kept building
until it led to the French Revolution. During this time of struggle and violence (1789-1799), thousands of
aristocrats, as well as the king and queen themselves, lost their lives at the guillotine. Perhaps if Louis XVI and
Marie Antoinette had reined in their extravagant spending, the events that rocked France would not have
occurred.
The people surrounding the queen encouraged her to spend money ____.
A. wisely
B. abundantly
C. carefully
D. foolishly
E. joyfully
39. The minister of finance tried to curb these royal ___.
A. aristocrats
B. money wasters
C. enemies
D. individuals
E. spenders
40. Questions 40-45.
Many great inventions are greeted with ridicule and disbelief. The invention of the airplane was no exception.
Although many people who heard about the first powered flight on December 17,1903, were excited and
impressed, others reacted with peals of laughter. The idea of flying an aircraft was repulsive to some people.
Such people called Wilbur and Orville Wright, the inventors of the first flying machine, impulsive fools.
Negative reactions, however, did not stop the Wrights. Impelled by their desire to succeed, they continued their
experiments in aviation.
Orville and Wilbur Wright had always had a compelling interest in aeronautics and mechanics. As young boys
they earned money by making and selling kites and mechanical toys. Later, they designed a newspaper-folding
machine, built a printing press, and operated a bicycle-repair shop. In 1896, when they read about the death of
Otto Lilienthal, the brother's interest in flight grew into a compulsion.
Lilienthal, a pioneer in hang-gliding, had controlled his gliders by shifting his body in the desired direction.
This idea was repellent to the Wright brothers, however, and they searched for more efficient methods to
control the balance of airborne vehicles. In 1900 and 1901, the Wrights tested numerous gliders and developed
control techniques. The brothers' inability to obtain enough lift power for the gliders almost led them to
abandon their efforts.
After further study, the Wright brothers concluded that the published tables of air pressure on curved surfaces
must be wrong. They set up a wind tunnel and began a series of experiments with model wings. Because of their
efforts, the old tables were repealed in time and replaced by the first reliable figures for air pressure on curved
surfaces. This work, in turn, made it possible for them to design a machine that would fly. In 1903 the Wrights
built their first airplane, which cost less than one thousand dollars. They even designed and built their own
source of propulsion- a lightweight gasoline engine. When they started the engine on December 17, the airplane
pulsated wildly before taking off. The plane managed to stay aloft for twelve seconds, however, and it flew one
hundred twenty feet.
By 1905 the Wrights had perfected the first airplane that could turn, circle, and remain airborne for half an hour
at a time. Others had flown in balloons or in hang gliders, but the Wright brothers were the first to build a full-
size machine that could fly under its own power. As the contributors of one of the most outstanding engineering
achievements in history, the Wright brothers are accurately called the fathers of aviation.
The idea of flying an aircraft was ___to some people.
A. boring
B. distasteful
C. exciting
D. needless
E. answer not available
41. People thought that the Wright brothers had ____.
A. acted without thinking
B. been negatively influenced
C. been too cautious
D. had not given enough thought
E. acted in a negative way
42. The Wright's interest in flight grew into a ____.
A. financial empire
B. plan
C. need to act
D. foolish thought
E. answer not in article
43. Lilenthal's idea about controlling airborne vehicles was ___the Wrights.
A. proven wrong by
B. opposite to the ideas of
C. disliked by
D. accepted by
E. opposed by
44. The old tables were __ and replaced by the first reliable figures for air pressure on curved surfaces.
A. destroyed
B. canceled
C. multiplied
D. discarded
E. not used
45. The Wrights designed and built their own source of ____.
A. force for moving forward
B. force for turning around
C. turning
D. force to going backward
E. none of the above
Answer Key
1. B
2. A
3. A
4. C
5. C
6. B
7. A
8. A
9. B
10. B
11. A
12. A
13. C
14. C
15. B
16. A
17. B
18. B
19. C
20. A
21. B
22. B
23. C
24. A
25. B
26. C
27. B
28. B
29. B
30. B
31. A
32. A
33. B
34. A
35. A
36. C
37. B
38. B
39. B
40. B
41. A
42. C
43. C
44. B
45. A
Practice Questions: Reading for the main idea
1. Americans have always been interested in their Presidents' wives. Many First Ladies have been remembered
because of the ways they have influenced their husbands. Other First Ladies have made the history books on
their own.
At least two First Ladies, Bess Truman and Lady Bird Johnson, made it their business to send signals during
their husbands' speeches. When Lady Bird Johnson thought her husband was talking too long, she wrote a note
and sent it up to the platform. It read, “It's time to stop!” And he did. Once Bess Truman didn't like what her
husband was saying on television, so she phoned him and said,” If you can't talk more politely than that in
public, you come right home.”
Abigail Fillmore and Eliza Johnson actually taught their husbands, Millard Fillmore and Andrew Johnson, the
thirteenth and seventeenth Presidents. A schoolteacher, Abigail eventually married her pupil, Millard. When
Eliza Johnson married Andrew, he could not read or write, so she taught him herself.
It was First Lady Helen Taft's idea to plant the famous cherry trees in Washington, D. C. Each spring these
blossoming trees attract thousands of visitors to the nation's capital. Mrs. Taft also influenced the male members
of her family and the White House staff in a strange way: she convinced them to shave off their beards!
Shortly after President Wilson suffered a stroke, Edith Wilson unofficially took over most of the duties of the
Presidency until the end of her husband's term. Earlier, during World War I, Mrs. Wilson had had sheep brought
onto the White House lawn to eat the grass. The sheep not only kept the lawn mowed but provided wool for an
auction sponsored by the First Lady. Almost $100,000 was raised for the Red Cross.
Dolly Madison saw to it that a magnificent painting of George Washington was not destroyed during the War of
1812. As the British marched toward Washington, D. C., she remained behind to rescue the painting, even after
the guards had left. The painting is the only object from the original White House that was not burned.
One of the most famous First Ladies was Eleanor Roosevelt, the wife of President Franklin D. Roosevelt. She
was active in political and social causes throughout her husband's tenure in office. After his death, she became
famous for her humanitarian work in the United Nations. She made life better for thousands of needy people
around the world.
What is the main idea of this passage?
A. The Humanitarian work of the First Ladies is critical in American government.
B. Dolly Madison was the most influential president's wife.
C. Eleanor Roosevelt transformed the First Lady image.
D. The First Ladies are important in American culture.
E. The First Ladies are key supporters of the Presidents.
2. Of the many kinds of vegetables grown all over the world, which remains the favorite of young and old alike?
Why, the potato, of course.
Perhaps you know them as “taters,” “spuds,” or “Kennebees,” or as “chips,” “Idahoes,” or even “shoestrings.”
No matter, a potato by any other name is still a potato- the world's most widely grown vegetable. As a matter of
fact, if you are an average potato eater, you will put away at least a hundred pounds of them each year.
That's only a tiny portion of the amount grown every year, however. Worldwide, the annual potato harvest is
over six billion bags- each bag containing a hundred pounds of spuds, some of them as large as four pounds
each. Here in the United States, farmers fill about four hundred million bags a year. That may seem like a lot of
“taters,” but it leaves us a distant third among world potato growers. Polish farmers dig up just over 800 million
bags a year, while the Russians lead the world with nearly 1.5 billion bags.
The first potatoes were grown by the Incas of South America, more than four hundred years ago. Their
descendants in Ecuador and Chile continue to grow the vegetable as high as fourteen thousand feet up in the
Andes Mountains. ( That's higher than any other food will grow.) Early Spanish and English explorers shipped
potatoes to Europe, and they found their way to North America in the early 1600s.
People eat potatoes in many ways-baked, mashed, and roasted, to name just three. However, in the United
States most potatoes are devoured in the form of French fries. One fast-food chain alone sells more than $1
billion worth of fries each year. No wonder, then, that the company pays particular attention to the way its fries
are prepared.
Before any fry makes it to the people who eat at these popular restaurants, it must pass many separate tests. Fail
any one and the spud is rejected. To start with, only russet Burbank potatoes are used. These Idaho potatoes
have less water content than other kinds, which can have as much as eighty percent water. Once cut into
“shoestrings” shapes, the potatoes are partly fried in a secret blend of oils, sprayed with liquid sugar to brown
them, steam dried at high heat, then flash frozen for shipment to individual restaurants.
Before shipping, though, every shoestring is measured. Forty percent of a batch must be between two and three
inches long. Another forty percent has to be over three inches. What about the twenty percent that are left in the
batch? Well, a few short fries in a bag are okay, it seems.
So, now that you realize the enormous size and value of the potato crop, you can understand why most people
agree that this part of the food industry is no “small potatoes.”
What is the main idea of this passage?
A. Potatoes from Ireland started the Potato Revolution.
B. The average American eats 50 lbs of potatoes a year.
C. French fries are made from potatoes.
D. Potatoes are a key vegetable in America.
E. The various terms for potatoes have a long history.
3. What does the word patent mean to you? Does it strike you as being something rather remote from your
interests? If it does, stop and think a moment about some of the commonplace things that you use every day,
objects that you take for granted as part of the world around you. The telephone, radio, television, the
automobile, and the thousand and one other things (even the humble safety pin) that enrich our lives today once
existed only as ideas in the minds of men. If it had not been possible to patent their ideas and thus protect them
against copying by others, these inventions might never have been fully developed to serve mankind.
If there were no patent protection there would be little incentive to invent and innovate, for once the details of
an invention became known, hordes of imitators who did not share the inventor's risks and expenses might well
flood the market with their copies of his product and reap much of the benefit of his efforts. The technological
progress that has made America great would wither rapidly under conditions such as these.
The fundamental principles in the U. S. patent structure came from England. During the glorious reign of Queen
Elizabeth I in England, the expanding technology was furthered by the granting of exclusive manufacturing and
selling privileges to citizens who had invented new processes or tools- a step that did much to encourage
creativity. Later, when critics argued that giving monopoly rights to one person infringed on the rights of others,
an important principle was added to the patent structure: The Lord Chief Justice of England stated that society
had everything to gain and nothing to lose by granting exclusive privileges to an inventor, because a patent for
an invention was granted for something new that society never had before.
Another basic principle was brought into law because certain influential people in England had managed to
obtain monopoly control over such age-old products as salt, and had begun charging as much as the traffic
would bear. The public outcry became so great that the government was forced to decree that monopoly rights
could be awarded only to those who created or introduced something really unique. These principles are the
mainstays of our modern patent system in the United States.
In colonial times patent law was left up to the separate states. The inconsistency, confusion, and unfairness that
resulted clearly indicated the need for a uniform patent law, and the men who drew up the Constitution
incorporated one. George Washington signed the first patent law on April 10,1790, and less than four months
later the first patent was issued to a man named Samuel Hopkins for a chemical process, an improved method of
making potash for use in soapmaking.
In 1936 the Patent Office was established as a separate bureau. From the staff of eight that it maintained during
its first year of operation it has grown into an organization of over 2500 people handling more than 1600 patent
applications and granting over 1000 every week.
The Patent Office in Washington, D. C., is the world's largest library of scientific and technical data, and this
treasure trove of information is open for public inspection. In addition to more than 3 million U. S. patents, it
houses more than 7 million foreign patents and thousands of volumes of technical literature. Abraham Lincoln
patented a device to lift steam vessels over river shoals, Mark Twain developed a self-pasting scrapbook, and
millionaire Cornelius Vanderbilt invented a shoe-shine kit.
A patent may be granted for any new and useful process, machine, article of manufacture, or composition of
matter ( a chemical compound or combinations of chemical compounds), or any distinct and new variety; of
plant, including certain mutants and hybrids.
The patent system has also helped to boost the wages of the American worker to an unprecedented level; he can
produce more and earn more with the computer, adding machines, drill press or lathe. Patented inventions also
help keep prices down by increasing manufacturing efficiency and by stimulating the competition that is the
foundation of our free enterprise system.
The decades of history have disclosed little need for modification of the patent structure. Our patent laws, like
the Constitution from which they grew, have stood the test of time well. They encouraged the creative
processes, brought untold benefits to society as a whole, and enabled American technology to outstrip that of
the rest of the civilized world.
What is the main idea of this passage?
A. The patent system encourages free enterprise.
B. The Constitution protects the patent system.
C. The patent system in England has been influential in American patent development.
D. Patents are important tools for inventors.
E. Patented inventions protect the inventor, free enterprise, and the creative process.
4. Most people think it's fine to be “busy as a beaver.” Little do they know. Beavers may work hard, but often
they don't get much done.
Beavers are supposed to be great tree cutters. It is true that a beaver can gnaw through a tree very quickly. (A
six-inch birch takes about ten minutes.) But then what? Often the beaver does not make use of the tree. One
expert says that beavers waste one out of every five trees they cut.
For one thing, they do not choose their trees wisely. One bunch of beavers cut down a cottonwood tree more
than one hundred feet tall. Then they found that they could not move it.
In thick woods a tree sometimes won't fall down. It gets stuck in the other trees. Of course, doesn't think to cut
down the trees that are in the way. So a good tree goes to waste.
Some people think that beavers can make a tree fall the way they want it to. Not true. (In fact, a beaver
sometimes gets pinned under a falling tree.) When beavers cut a tree near a stream, it usually falls into the
water. But they do not plan it that way. The fact is that most trees lean toward the water to start with.
Now what about dam building? Most beaver dams are wonders of engineering. The best ones are strongly built
of trees, stones, and mud. They are wide at the bottom and narrow at the top.
Beavers think nothing of building a dam more than two hundred feet long. One dam, in Montana, was more
than two thousand feet long. The largest one ever seen was in New Hampshire. It stretched four thousand feet. It
made a lake large enough to hold forty beaver homes.
So beavers do build good dams. But they don't always build them in the right places. They just don't plan. They
will build a dam across the widest part of the stream. They don't try to find a place where the stream is narrow.
So a lot of their hard work is wasted.
Beavers should learn that it's not enough to be busy. You have to know what you're doing, too. For example,
there was one Oregon beaver that really was a worker. It decided to fix a leak in a man-made dam. After five
days of work it gave up. The leak it was trying to block was the lock that boats go through.
What is the main idea of this passage?
A. Beavers may be hard working animals, but they don't always choose the most efficient mechanisms.
B. Beavers are excellent dam builders.
C. New Hampshire was the site of the largest beaver dam.
D. Beavers are well developed tree cutters.
E. Beavers are poor surveyors of aquatic environments in some cases.
5. The raisin business in America was born by accident. It happened in 1873 in the San Joaquin Valley of
California. Many farmers raised grapes in this valley. That year, just before the grape harvest, there was a heat
wave. It was one of the worst heat waves ever known. It was so hot the grapes dried on the vines. When they
were picked, California had its first raisin crop.
People were surprised to find how good raisins were. Everybody wanted more. So the San Joaquin farmers went
into the raisin business. Today, of course, they do not let the grapes dry on the vines. They treat them with much
more care.
In late August the grapes start to ripen. They are tested often for sweetness. The growers wait until the sugar
content is twenty-one percent. Then they know the grapes are ripe enough to be picked.
Skilled workers come to the vineyards. They pick the bunches of grapes by hand. The workers fill their flat pans
with grapes. They gently empty the pans onto squares of paper. These squares lie between the long rows of
vines. They sit in the sun.
Here the grapes stay while the sun does its work. It may take two weeks or longer. The grapes are first dried on
one side. When they have reached the right color, they are turned to dry on the other side. The grapes are dried
until only fifteen percent of the moisture is left. Then they have turned into raisins.
The raisins are rolled up in the paper on which they have dried. Trucks take them from the fields. They are
poured into big boxes called sweatboxes. Each box holds one hundred and sixty pounds of raisins. Here, any
raisins that are a bit too dry take moisture from those that have a bit too much. After a while they are all just
moist enough.
The big boxes are trucked next to the packaging plant. They are emptied onto a conveyor belt that shakes the
raisins gently. This knocks them from their stems. A blast of air whisks the stems away. The water bath is next.
Then the plump brown raisins have a last inspection. They are again checked for moisture and sugar. Then they
go on a belt to packing machines. Here they are poured into packages, which are automatically weighed and
sealed. The raisins are now ready for market.
What is the main idea of this passage?
A. The creation of raisins in America was an accident.
B. The process of raisin development requires multiple steps.
C. Raisins on the grocery store shelf undergo a brief fermentation process.
D. Raisins are cleaned thoroughly at the packing plant.
E. California has been the leader in American raisin development.
6. In 1976, Sichan Siv was crawling through the jungle, trying to escape from Cambodia. By 1989, however,
Siv was working in the White House, in Washington D. C., as an advisor to the President of the United States.
How did this strange journey come about?
Like millions of Cambodians, Siv was a victim of a bloody civil war. One of the sides in this war was the
Cambodian government. The other was a group called the Khmer Rouge. When the Khmer Rouge won the war,
the situation in Cambodia got worse. Many people were killed, while others were forced into hard labor.
Sometimes entire families were wiped out.
Siv came from a large family that lived in the capital of Cambodia. After finishing high school, Siv worked for
a while with a Cambodian airline company. Later, he taught English. After that, he took a job with CARE, an
American group that was helping victims of the war.
Siv had hope to leave Cambodia before the Khmer Rouge took over the country. Unfortunately, he was delayed.
As a result, he and his family were taken from their homes and forced to labor in rice fields. After a while, Siv
managed to escape. He rode an old bicycle for miles, trying to reach Thailand where he would be free and safe.
For three weeks he slept on the ground and tried to hide from the soldiers who were looking for him. Caught at
last, he was afraid he would be killed. Instead, he was put into a labor camp, where he worked eighteen hours
each day without rest. After several months, he escaped again; this time he made it. The journey, however, was
a terrifying one. After three days of staggering on foot through mile after mile of thick bamboo, Siv finally
made his way to Thailand.
Because he had worked for an American charity group, Siv quickly found work in a refugee camp. Soon he was
on his way to the states. He arrived in June of 1976 and got a job-first picking apples and then cooking in a fastfood restaurant. Siv, however, wanted more than this; he wanted to work with people who, like himself, had
suffered the hardship of leaving their own countries behind. Siv decided that the best way to prepare for this
kind of work was to go to college. He wrote letters to many colleges and universities. They were impressed with
his school records from Cambodia, and they were impressed with his bravery. Finally, in 1980, he was able to
study at Columbia University in New York City. After finishing his studies at Columbia, Siv took a job with the
United Nations. He married an American woman and became a citizen. After several more years, he felt that he
was very much a part of his new country.
In 1988, Siv was offered a job in the White House working for President Reagan's closest advisors. It was a
difficult job, and he often had to work long hours. However the long hard work was worth it, because Siv got
the opportunity to help refugees in his work.
What is the main idea of this passage?
A. Persistence and courage are global ideas.
B. Siv covered a large area during his life.
C. Siv persevered to become an American citizen
D. Siv overcame numerous challenges to come to American and help others.
E. Siv persevered to become an American citizen.
7. When you want to hang the American flag over the middle of a street, suspend it vertically with the blue
field, called the union, to the north and east-west street. When the flag is displayed with another banner from
crossed staffs, the American flag is on the right. Place the staff of the American flag in front of the other staff.
Raise the flag quickly and lower it slowly and respectfully. When flying the flag at half-mast, hoist it to the top
of the pole for a moment before lowering it to mid-pole. When flying the American flag with banners from
states or cities, raise the nation's banner first and lower it last. Never allow the flag to touch the ground.
What is the main idea of this passage?
A. The American flag is the symbol of American freedom.
B. The American flag has fifty stars.
C. Placing the American flag inappropriately will draw government intervention.
D. American flag should be flown differently in certain situations.
","The flag should be lowered quickly and respectfully.
8. What if someone told you about a kind of grass that grows as tall as the tallest trees? A grass that can be
made as strong as steel? A grass from which houses, furniture, boats, and hundreds of other useful things can be
made? A grass that you would even enjoy eating? Would you believe that person? You should, for that grass is
bamboo, the “wood” of 1,001 uses.
Bamboo may look like wood, but it is part of the family of plants that includes wheat, oats, and barley. It is a
kind of grass. This grass is not just a material for making useful products. Young bamboo is eaten, often mixed
with other vegetables, in many Asian foods.
Bamboo grows in many parts of the world. In the United States it grows in an area from Virginia west to
Indiana and south to Florida, Louisiana, and Texas. Most bamboo, however, is found in warm, wet climates,
especially in Asia and on the islands of the South Pacific Ocean.
In most Asian countries, bamboo is nearly as important as rice. Many Asians live in bamboo houses. They sit on
bamboo chairs and sleep on bamboo mats. They fence their land with bamboo and use the wood for cages for
chickens and pigs.
Bamboo is used to build large buildings as well as homes. When it is glued in layers, it becomes as strong as
steel. On some islands in the South Pacific, bamboo is even used for water pipes. This extraordinary material
has many other uses. It is used to make musical instruments, such as flutes and recorders. Paper made from
bamboo has been highly prized by artists for thousands of years.
Bamboo is light and strong, and it bends without breaking. It is cheap, floats on water, almost never wears out,
and is easy to grow. Nothing else on earth grows quite so fast as bamboo. At times you can even see it grow!
Botanists have recorded growths of more than three feet in just twenty-four hours! Bamboo is hollow and has a
strong root system that almost never stops growing and spreading. In fact, only after it flowers, an event that
may happen only once every thirty years, will bamboo die.
There are more than a thousand kinds of bamboo. The smallest is only three inches tall and one-tenth of an inch
across. The largest reaches more than two hundred feet in height and seven inches in diameter. No wonder,
then, that the lives of nearly half the people on earth would change enormously if there were no longer any
bamboo. No wonder, too, that to many people bamboo is a symbol of happiness and good fortune.
What is the main idea of this passage?
A. Bamboo has at least 2,000 uses.
B. Bamboo grows at an amazing rate and is found primarily in Asia.
C. Bamboo is an amazing grass that can be used in multiple ways.
D. There are at least a 1,000 types of bamboo.
E. Bamboo could be considered a flower in some cases.
9. Every year since 1986, some of the world's most daring runners have gathered in the desert of Morocco. They
are there to take part in one of the most difficult races in the world. The Marathon of the Sands, as it is called,
covers over 125 miles of desert and mountain wilderness. The runners complete the course in fewer than seven
days, and they run with their food, clothing, and sleeping bags on their backs.
The Marathon of the Sands was founded in 1986 by Patrick Bauer. His idea was to give the runners, who come
from all over the world, a special kind of adventure. Most of the runners in this race have found that they form
deep friendships with the other runners during their days and nights in the desert. Facing terrible heat and
complete exhaustion, they learn much about themselves and each other.
For most of the runners, though, the challenge of the race is the main reason for coming. On the first day, for
example, they run fifteen miles across a desert of sand, rocks, and thorny bushes. Few runners finish the day
without blistered and raw feet. They also suffer from a lack of water. (They are allowed less than nine quarts of
water during each day of the race.) Most of all, they are exhausted when they arrive at the campsite for the
night.
The second day, the runners are up at 6:00 A. M. Within a few hours, it is 100 degrees F, but the runners do not
hesitate. They must cover eighteen miles that day. That night, they rest. They must be ready for the next day's
run.
On the third day, the runners must climb giant sand dunes- the first they have faced. Dust and sand mix with the
runners' sweat. Soon their faces are caked with mud. After fifteen miles of these conditions, the runners finally
reach their next camp.
The race continues like this for four more days. The fourth and fifth days are the worst. On the fourth day, the
runners pass through a level stretch and a beautiful, tree-filled oasis, but then, on this and on the next day, they
cross more than twenty-one miles of rocks and sand dunes. The temperature soars to 125 degrees F, and many
runners cannot make it. Helicopters rush fallen runners to medical help. Runners who make it to the end of the
fifth day know that the worst is over.
On the sixth day, heat and rocks punish the racers terribly. In the Valley of Dra, the wind picks up and, as the
desert heat is thrust against them with great force, they grow more and more exhausted.
The seventh day is the last, with only twelve miles to be covered. The dusty, tired, blistered runners set out at
daybreak. Near the finish line, children race along with the runners, for everybody has caught the excitement.
The ones who have run the whole marathon know they have accomplished what most people could not even
dream of. “During the hard moments,” says one contestant who has raced here twice, “I'd think, ‘Why am I
here?' Then I'd realize I was there to find my limits.”
What is the main idea of this passage?
A. The Marathon of the Sands race tests the limits of human endurance.
B. The runners run at their own pace.
C. The race causes the strong to stumble and the weak to not finish.
D. The seventh day is the hardest day of the race.
E. Every runner runs the race to find their human limits.
10. High in the Andes Mountains in Peru stands the ancient city of Machu Picchu. No one knows why this great
city was built, nor is it likely that we will ever know. Nevertheless, the deserted city of Machu Picchu is
important for what it reveals about the ancient Inca people of South America.
The Incas once ruled a great empire that covered a large part of the South American continent. The empire was
more than five hundred years old when the first Spanish explorers, looking for gold, went to that continent in
the sixteenth century.
The Incas were an advanced people. They were skillful engineers who paved their roads and built sturdy
bridges. They plowed the land in such a way that rains would not wash away valuable soil. They dug ditches to
carry water into dry areas for farming.
Even though they did not know about the wheel, the Incas were able to move huge stone blocks- some as heavy
as ten tons- up the sides of mountains to build walls. The blocks were fitted so tightly, without cement of any
kind, that it would be impossible to slip a knife blade between them! The walls have stood firm through great
storms and earthquakes that have destroyed many modern buildings.
The Incas were great artists, too. Today, Incan dishes and other kinds of pottery are prized for their wonderful
designs. Since both gold and silver were in great supply, the Incas created splendid objects from these precious
metals.
While it is true that the Incas had no written language, they kept their accounts by using a system of knotted
strings of various lengths and colors. The sizes of the knots and the distances between them represented
numbers.
At its height, the Incan empire included as many as thirty million people. The emperor ruled them with an iron
hand. He told his subjects where to live, what to plant, how long they should work-even whom they could
marry. Since he owned everything, the emperor gave what he wished when he wished- and in the amount he
wished -to his people.
In 1533 Spanish explorers led by Francisco Pizarro murdered the emperor of the Incas. Earlier, the heir to the
Incan empire had also been killed. The Incas, who had always been entirely dependent on their emperor, now
had no recognized leader. The Spaniards easily conquered the empire and plundered its riches.
Have the Incas disappeared from South America? Not at all. In Peru alone, once the center of that great empire,
eighty percent of the twenty million people are descendants of the Inca people. Evidence of the Incan empire
can be found in many other places in South America as well. You can even visit Machu Picchu. The remains of
this ancient city still stand high in the mountains of Peru, an awesome tribute to this once powerful empire.
What is the main idea of this passage?
A. The Incas once inhabited the ancient city of Machu Picchu.
B. Peru was the primary country of the Incas.
C. The Incan empire can be found in ancient cities and was plundered by the Spanish.
D. Spanish conquerors destroyed the Incan empire in the thirteenth century.
E. Machu Picchu was the capital of the Incan empire.
Answer Key
1. D
2. D
3. E
4. A
5. B
6. D
7. D
8. C
9. A
10. C
Practice Questions: Advanced Reading Comprehension
1. In 1892 the Sierra Club was formed. In 1908 an area of coastal redwood trees north of San Francisco was
established as Muir Woods National Monument. In the Sierra Nevada mountains, a walking trail from Yosemite
Valley to Mount Whitney was dedicated in 1938. It is called John Muir Trail.
John Muir was born in 1838 in Scotland. His family name means “moor,” which is a meadow full of flowers
and animals. John loved nature from the time he was small. He also liked to climb rocky cliffs and walls.
When John was eleven, his family moved to the United States and settled in Wisconsin. John was good with
tools and soon became an inventor. He first invented a model of a sawmill. Later he invented an alarm clock
that would cause the sleeping person to be tipped out of bed when the timer sounded.
Muir left home at an early age. He took a thousand-mile walk south to the Gulf of Mexico in 1867and 1868.
Then he sailed for San Francisco. The city was too noisy and crowded for Muir, so he headed inland for the
Sierra Nevadas.
When Muir discovered the Yosemite Valley in the Sierra Nevadas, it was as if he had come home. He loved the
mountains, the wildlife, and the trees. He climbed the mountains and even climbed trees during thunderstorms
in order to get closer to the wind. He put forth the theory in the late 1860's that the Yosemite Valley had been
formed through the action of glaciers. People ridiculed him. Not until 1930 was Muir's theory proven correct.
Muir began to write articles about the Yosemite Valley to tell readers about its beauty. His writing also warned
people that Yosemite was in danger from timber mining and sheep ranching interests. In 1901 Theodore
Roosevelt became president of the United States. He was interested in conservation. Muir took the president
through Yosemite, and Roosevelt helped get legislation passed to create Yosemite National Park in 1906.
Although Muir won many conservation battles, he lost a major one. He fought to save the Hetch Valley, which
people wanted to dam in order to provide water for San Francisco. In the late 1913 a bill was signed to dam the
valley. Muir died in 1914. Some people say losing the fight to protect the valley killed Muir.
What happened first?
A. The Muir family moved to the United States.
B. Muir Woods was created.
C. John Muir learned to climb rocky cliffs.
D. John Muir walked to the Gulf of Mexico
E. Muir visited along the east coast.
2. When did Muir invent a unique form of alarm clock?
A. while the family still lived in Scotland
B. after he sailed to San Francisco
C. after he traveled in Yosemite
D. while the Muir family lived in Wisconsin
E. after he took the long walk
3. What did John Muir do soon after he arrived in San Francisco?
A. He ran outside during an earthquake.
B. He put forth a theory about how Yosemite was formed.
C. He headed inland for the Sierra Nevadas.
D. He began to write articles about the Sierra Nevadas.
E. He wrote short stories for the local newspaper.
4. When did John Muir meet Theodore Roosevelt?
A. between 1901 and 1906
B. between 1838 and 1868
C. between 1906 and 1914
D. between 1868 and 1901
E. between 1906-1907
5. What happened last?
A. John Muir died.
B. John Muir Trail was dedicated.
C. Muir's glacial theory was proven.
D. The Sierra Club was formed.
E. John's family visited him.
6. When using a metal file, always remember to bear down on the forward stroke only. On the return stroke, lift
the file clear of the surface to avoid dulling the instrument's teeth. Only when working on very soft metals is it
advisable to drag the file's teeth slightly on the return stroke. This helps clear out metal pieces from between the
teeth.
It is best to bear down just hard enough to keep the file cutting at all times. Too little pressure uses only the tips
of the teeth; too much pressure can chip the teeth. Move the file in straight lines across the surface. Use a vice to
grip the work so that your hands are free to hold the file. Protect your hands by equipping the file with a handle.
Buy a wooden handle and install it by inserting the pointed end of the file into the handle hole.
These directions show you how toA. work with a hammer
B. use a file
C. polish a file
D. oil a vise
E. repair shop tools
7. When using a fileA. always bear down on the return stroke
B. move it in a circle
C. remove the handle
D. press down on the forward stroke
E. wear protective gloves
8. When working on soft metals, you canA. remove the handle
B. clear metal pieces from the teeth
C. bear down very hard on the return stroke
D. file in circles
E. strengthen them with added wood
9. Protect your hands byA. dulling the teeth
B. dragging the teeth on the backstroke
C. using a vise
D. installing a handle
E. wearing safety gloves
10. “Old woman,” grumbled the burly white man who had just heard Sojourner Truth speak, “do you think your
talk about slavery does any good? I don't care any more for your talk than I do for the bite of a flea.”
The tall, imposing black woman turned her piercing eyes on him. “Perhaps not,” she answered, “but I'll keep
you scratching.”
The little incident of the 1840s sums up all that Sojourner Truth was: utterly dedicated to spreading her
message, afraid of no one, forceful and witty in speech.
Yet forty years earlier, who could have suspected that a spindly slave girl growing up in a damp cellar in upstate
New York would become one of the most remarkable women in American history? Her name then was Isabella
(many slaves had no last names), and by the time she was fourteen she had seen both parents die of cold and
hunger. She herself had been sold several times. By 1827, when New York freed its slaves, she had married and
borne five children.
The first hint of Isabella's fighting spirit came soon afterwards, when her youngest son was illegally seized and
sold. She marched to the courthouse and badgered officials until her son was returned to her.
In 1843, inspired by religion, she changed her name to Sojourner(meaning “one who stays briefly”) Truth, and,
with only pennies in her purse, set out to preach against slavery. From New England to Minnesota she trekked,
gaining a reputation for her plain but powerful and moving words. Incredibly, despite being black and female
(only white males were expected to be public speakers), she drew thousands to town halls, tents, and churches
to hear her powerful, deep-voiced pleas on equality for blacks-and for women. Often she had to face threatening
hoodlums. Once she stood before armed bullies and sang a hymn to them. Awed by her courage and her
commanding presence, they sheepishly retreated.
During the Civil War she cared for homeless ex-slaves in Washington. President Lincoln invited her to the
White House to bestow praise on her. Later, she petitioned Congress to help former slaves get land in the West.
Even in her old age, she forced the city of Washington to integrate its trolley cars so that black and white could
ride together.
Shortly before her death at eighty-six, she was asked what kept her going. “I think of the great things,” replied
Sojourner.
The imposing black woman promised to keep the white manA. searching
B. crying
C. hollering
D. scratching
E. fleeing
11. This incident occurred in theA. 1760s
B. 1900s
C. 1840s
D. 1920s
E. 1700s
12. Sojourner Truth was raised in a damp cellar inA. New York
B. Georgia
C. New Jersey
D. Idaho
E. Maryland
13. Isabella lost both parents by the time she wasA. twenty-seven
B. two
C. seven
D. fourteen
E. nineteen
14. When New York freed its slaves, Isabella had-
A. problems
B. no children
C. five children
D. an education
E. three children
15. Her change in name was inspired byA. a fighting spirit
B. religion
C. her freedom
D. officials
E. friends
16. She traveled from New England toA. Canada
B. California
C. Minnesota
D. Alaska
E. Virginia
17. She forced the city of Washington toA. integrate its trolleys
B. give land grants
C. care for ex-slaves
D. provide food for ex-slaves
E. clean its trolleys
18. She preached againstA. smoking
B. slavery
C. alcohol
D. hoodlums
E. women having no rights
19. Sojourner Truth died atA. 48
B. 72
C. 63
D. 86
E. 88
20. The Galapagos Islands are in the Pacific Ocean, off the western coast of South America. They are a rocky,
lonely spot, but they are also one of the most unusual places in the world. One reason is that they are the home
of some of the last giant tortoises left on earth.
Weighing hundreds of pounds, these tortoises, or land turtles, wander slowly around the rocks and sand of the
islands. Strangely, each of these islands has its own particular kinds of tortoises. There are seven different kinds
of tortoises on the eight islands, each kind being slightly different from the other.
Hundreds of years ago, thousands of tortoises wandered around these islands. However, all that changed when
people started landing there. When people first arrived in 1535, their ships had no refrigerators. This meant that
fresh food was always a problem for the sailors on board. The giant tortoises provided a solution to this
problem.
Ships would anchor off the islands, and crews would row ashore and seize as many tortoises as they could.
Once the animals were aboard the ship, the sailors would roll the tortoises onto their backs. The tortoises were
completely helpless once on their backs, so they could only lie there until used for soups and stews. Almost
100,000 tortoises were carried off in this way.
The tortoises faced other problems, too. Soon after the first ships, settlers arrived bringing pigs, goats, donkeys,
dogs and cats. All of these animals ruined life for the tortoises. Donkey and goats ate all the plants that the
tortoises usually fed on, while the pigs. Dogs and cats consumed thousands of baby tortoises each year. Within
a few years, it was hard to find any tortoise eggs-or even any baby tortoises.
By the early 1900s, people began to worry that the last of the tortoises would soon die out. No one, however,
seemed to care enough to do anything about the problem. More and more tortoises disappeared, even though
sailors no longer needed them for food. For another fifty years, this situation continued. Finally, in the 1950s,
scientist decided that something must be done.
The first part of their plan was to get rid of as many cats, dogs and other animals as they could. Next, they tried
to make sure that more baby tortoises would be born. To do this, they started looking for wild tortoise eggs.
They gathered the eggs and put them in safe containers. When the eggs hatched, the scientists raised the
tortoises in special pens. Both the eggs and tortoises were numbered so that the scientists knew exactly which
kinds of tortoises they had-and which island they came from. Once the tortoises were old enough and big
enough to take care of themselves, the scientists took them back to their islands and set them loose. This slow,
hard work continues today, and, thanks to it, the number of tortoises is now increasing every year. Perhaps these
wonderful animals will not disappear after all.
What happened first?
A. Sailors took tortoises aboard ships.
B. The tortoise meat was used for soups and stews.
C. Tortoises were put onto their backs.
D. Settlers brought other animals to the islands.
E. Pigs had been all the sailors had to eat.
21. What happened soon after people brought animals to the islands?
A. Tortoise eggs were kept in safe containers.
B. Scientists took away as many animals as they could.
C. The animals ate the tortoises' food and eggs.
D. The tortoises fought with the other animals.
E. The tortoises continued to wander freely.
22. When did people start to do something to save the tortoises?
A. in the 1500s
B. in the 1950s
C. in the early 1900s
D. in the 1960s
E. in the 1400s
23. What happens right after the tortoise eggs hatch?
A. The scientists take the tortoises back to their islands.
B. The scientists get rid of cats, dogs, and other animals.
C. The sailors use the tortoises for food.
D. The scientist raised the tortoises in special pens.
E. The scientist encouraged the villagers to help.
24. What happened last?
A. The tortoises began to disappear.
B. The number of tortoises began to grow.
C. Scientists took away other animals.
D. Tortoises were taken back to their home islands.
E. The number of tortoises began to decrease.
25. The first person in the group starts off by naming anything that is geographical. It could be a city, state,
country, river, lake, or any proper geographical term. For example, the person might say,”Boston.” The second
person has ten seconds to think of how the word ends and come up with another geographical term starting with
that letter. The second participant might say, “Norway,” since the geographical term has to start with “N.” The
third person would have to choose a word beginning with “ Y.” If a player fails to think of a correct answer
within the time limit, that player is out of the game. The last person to survive is the champion.
This game may help you withA. history
B. music
C. geography
D. sports
E. current events
26. The person trying to answer needsA. no time limit
B. to know geography only
C. to ignore the last letters of words
D. to know something about spelling and geography
E. to be a good speller
27. Before you choose your own word, think about howA. the last word starts
B. the last word ends
C. smart you are
D. long the last word is
E. the spelling of the first word
28. The answer must beA. in New York
B. within the United States
C. proper geographical terms
D. in the same region
E. along a coast line
29. Charles A. Lindbergh is remembered as the first person to make a nonstop solo flight across the Atlantic, in
1927. This feat, when Lindbergh was only twenty-five years old, assured him a lifetime of fame and public
attention.
Charles Augustus Lindbergh was more interested in flying airplanes than he was in studying. He dropped out of
the University of Wisconsin after two years to earn a living performing daredevil airplane stunts at country
fairs. Two years later, he joined the United States Army so that he could go to the Army Air Service flighttraining school. After completing his training, he was hired to fly mail between St. Louis and Chicago.
Then came the historic flight across the Atlantic. In 1919, a New York City hotel owner offered a prize of
$25,000 to the first pilot to fly nonstop from New York to Paris. Nine St. Louis business leaders helped pay for
the plane Lindbergh designed especially for the flight. Lindbergh tested the plane by flying it from San Diego to
New York, with an overnight stop in St. Louis. The flight took only 20 hours and 21 minutes, a transcontinental
record.
Nine days later, on May 20,1927, Lindbergh took off from Long Island, New York, at 7:52 A. M. He landed at
Paris on May 21 at 10:21 P. M. He had flown more than 3,600 miles in less than thirty four hours. His flight
made news around the world. He was given awards and parades everywhere he went. He was presented with the
U. S. Congressional Medal of Honor and the first Distinguished Flying Cross. For a long time, Lindbergh
toured the world as a U. S. goodwill ambassador. He met his future wife, Anne Morrow, in Mexico, where her
father was the United States ambassador.
During the 1930s, Charles and Anne Lindbergh worked for various airline companies, charting new commercial
air routes. In 1931, for a major airline, they charted a new route from the east coast of the United States to the
Orient. The shortest, most efficient route was a great curve across Canada, over Alaska, and down to China and
Japan. Most pilots familiar with the Arctic did not believe that such a route was possible. The Lindberghs took
on the task of proving that it was. They arranged for fuel and supplies to be set out along the route. On July 29,
they took off from Long Island in a specially equipped small seaplane. They flew by day and each night landed
on a lake or a river and camped. Near Nome, Alaska, they had their first serious emergency. Out of daylight and
nearly out of fuel, they were forced down in a small ocean inlet. In the next morning's light, they discovered
they had landed on barely three feet of water. On September 19, after two more emergency landings and
numerous close calls, they landed in China with the maps for a safe airline passenger route.
Even while actively engaged as a pioneering flier, Lindbergh was also working as an engineer. In 1935, he and
Dr. Alexis Carrel were given a patent for an artificial heart. During World War I in the 1940s, Lindbergh served
as a civilian technical advisor in aviation. Although he was a civilian, he flew over fifty combat missions in the
Pacific. In the 1950s, Lindbergh helped design the famous 747 jet airliner. In the late 1960s, he spoke widely on
conservation issues. He died August 1974, having lived through aviation history from the time of the first
powered flight to the first steps on the moon and having influenced a big part of that history himself.
What did Lindbergh do before he crossed the Atlantic?
A. He charted a route to China.
B. He graduated from flight-training school.
C. He married Anne Morrow.
D. He acted as a technical advisor during World War II.
E. He was responsible for the fuel supply for planes.
30. What happened immediately after Lindbergh crossed the Atlantic?
A. He flew the mail between St. Louis and Chicago.
B. He left college.
C. He attended the Army flight-training school.
D. He was given the Congressional Medal of Honor.
E. He married Anne Morrow.
31. When did Charles meet Anne Morrow?
A. before he took off from Long Island
B. after he worked for an airline
C. before he was forced down in an ocean inlet
D. after he received the first Distinguished Flying Cross
E. when visiting his parents
32. When did the Lindberghs map an air route to China?
A. before they worked for an airline
B. before Charles worked with Dr. Carrel
C. after World War II
D. while designing the 747
E. when he was thirty
33. What event happened last?
A. Lindbergh patented an artificial heart.
B. The Lindberghs mapped a route to the Orient.
C. Lindbergh helped design the 747 airline.
D. Lindbergh flew fifty combat missions.
E. Charles finally was given an honorary degree from college.
34. Always read the meter dials from the right to the left. This procedure is much easier, especially if any of the
dial hands are near the zero mark. If the meter has two dials, and one is smaller than the other, it is not
imperative to read the smaller dial since it only registers a small amount. Read the dial at the right first. As the
dial turns clockwise, always record the figure the pointer has just passed. Read the next dial to the left and
record the figure it has just passed. Continue recording the figures on the dials from right to left. When finished,
mark off the number of units recorded. Dials on water and gas meters usually indicate the amount each dial
records.
These instructions show you how to –
A. read a meter
B. turn the dials of a meter
C. install a gas meter
D. repair a water meter
E. be prepared for outside employment
35. Always read the meter dials-
A. from top to bottom
B. from right to left
C. from left to right
D. from the small to the large dial
E. from the large dial to the small dial
36. As you read the first dial, record the figures
A. on the smaller dial
B. the pointer is approaching
C. the pointer has just passed
D. at the top
E. at the bottom
37. When you have finished reading the meter, mark offA. the number of units recorded
B. the figures on the small dial
C. the total figures
D. all the zero marks
E. the last reading of the month
38. The village of Vestmannaeyjar, in the far northern country of Iceland, is as bright and clean and up-to-date
as any American or Canadian suburb. It is located on the island of Heimaey, just off the mainland. One January
night in 1973, however, householders were shocked from their sleep. In some backyards red-hot liquid was
spurting from the ground. Flaming “skyrockets” shot up and over the houses. The island's volcano, Helgafell,
silent for seven thousand years, was violently erupting!
Luckily, the island's fishing fleet was in port, and within twenty-four hours almost everyone was ferried to the
mainland. But then the agony of the island began in earnest. As in a nightmare, fountains of burning lava
spurted three hundred feet high. Black, baseball-size cinders rained down. An evil-smelling, eye-burning,
throat-searing cloud of smoke and gas erupted into the air, and a river of lava flowed down the mountain. The
constant shriek of escaping steam was punctuated by ear-splitting explosions.
As time went on, the once pleasant village of Vestmannaeyjar took on a weird aspect. Its street lamps still
burning against the long Arctic night, the town lay under a thick blanket of cinders. All that could be seen above
the ten-foot black drifts were the tips of street signs. Some houses had collapsed under the weight of cinders;
others had burst into flames as the heat ignited their oil storage tanks. Lighting the whole lurid scene, fire
continued to shoot from the mouth of the looming volcano.
The eruption continued for six months. Scientists and reporters arrived from around the world to observe the
awesome natural event. But the town did not die that easily. In July, when the eruption ceased, the people of
Heimaey Island returned to assess the chances of rebuilding their homes and lives. They found tons of ash
covering the ground. The Icelanders are a tough people, however, accustomed to the strange and violent nature
of their Arctic land. They dug out their homes. They even used the cinders to build new roads and airport
runways. Now the new homes of Heimaey are warmed from water pipes heated by molten lava.
The village is located on the island ofA. Vestmannaeyjar
B. Hebrides
C. Heimaey
D. Helgafell
E. Heimma
39. The color of the hot liquid wasA. orange
B. black
C. yellow
D. red
E. gray
40. This liquid was coming from the –
A. mountains
B. ground
C. sea
D. sky
E. ocean
41. The island's volcano had been inactive forA. seventy years
B. seven thousand years
C. seven thousand months
D. seven hundred years
E. seventy decades
42. Black cinders fell that were the size of__
A. baseballs
B. pebbles
C. golf balls
D. footballs
E. hail-stones
43. Despite the eruptionA. buses kept running
B. the radio kept broadcasting
C. the police kept working
D. street lamps kept burning
E. the television kept broadcasting
44. This volcanic eruption lasted for six ___.
A. weeks
B. hours
C. months
D. days
E. years
Answer Key
1. C
2. D
3. C
4. A
5. B
6. B
7. D
8. B
9. D
10. D
11. C
12. A
13. D
14. C
15. B
16. C
17. A
18. B
19. D
20. A
21. C
22. B
23. D
24. B
25. C
26. D
27. B
28. C
29. B
30. D
31. D
32. B
33. C
34. A
35. B
36. C
37. A
38. C
39. D
40. B
41. B
42. A
43. D
44. C
Practice Questions: Sentence Correction
1. If the books have been cataloged last week, why haven't they been placed on the shelf?
A. have been cataloged
B. would have been cataloged
C. was cataloged
D. were cataloged
E. had been cataloged
2. Jessica Mitford wrote The American Way of Death, a best-selling book, that led eventually to an official
investigation of the funeral industry.
A. that led eventually
B. that had led eventually
C. that eventually led
D. which led eventually
E. who eventually led
3. Sabotage came from the French saboter, which means“to clatter with wooden shoes (sabots).”
A. which means “ to
B. which means, “ to
C. that means “to
D. that means- “to
E. that means, “to
4. In studying an assignment it is wise to read it over quickly at first, than see the major points, and finally
outline the material.
A. first, than
B. first: then
C. first-then
D. first, then
E. first-than
5. To judge the Tidy City contest, we picked an uninterested party.
A. picked an uninterested party.
B. picked an interested party!
C. picked a disinterested party.
D. are in the process of picking an uninterested party.
E. picked an disinterested party.
6. Linda decides they had better scram before the killers find them.
A. had better scram
B. had better leave
C. should hurry and scram
D. could hurry and leave
E. had better get out
7. I really dug the character of Brutus.
A. dug
B. thought about
C. thought of
D. admired
E. gazed at
8. Once upon a point a time, a small person named Little Red Riding Hood initated plans for the preparation,
delivery and transportation of foodstuffs to her Grandmother.
A. and transportation of foodstuffs to her Grandmother.
B. and transportation of food stuffs to her Grandmother.
C. and transportation of food supplies to her Grandmother.
D. and transportation of foodstuffs to her grandmother.
E. and, transportation of food supplies to her grand mother.
9. The setting of a story effects the story's plot.
A. effects the story's plot
B. effects the stories plot
C. affect the story's plot
D. affects the story's plot
E. affects the story's plots
10. Arctic trees are scrubbiest than trees in milder climates.
A. scrubbiest than trees
B. scrubbier then trees
C. scrubbiest than are trees
D. scrubbier than are trees
E. scrubbier than trees
11. Quebec rises in a magnificent way above the St. Lawrence River.
A. rises in a magnificent way above
B. rises in a magnificent way, way above
C. rises magnificently above
D. rises magnificently way above
E. is raised in a magnificent way above
12. Someone gives the school gerbils every year.
A. Someone gives the school gerbils
B. Some one gives the school gerbils
C. Some one gives the School gerbils
D. There is a person that gives the school gerbils
E. An individual gave gerbils
13. During colonial days, a school room looked rather empty.
A. colonial days, a school room looked
B. colonial days, a schoolroom looked
C. colonial days; a schoolroom looked
D. colonial days; a school room looked
E. colonial days- a schoolroom looked
14. The helium- filled balloon rose in the air.
A. rose in the air.
B. was rising in the air.
C. was in the air.
D. rose into the air.
E. would rise in the air.
15. If I had the address, I would have delivered the package myself.
A. had the address,
B. had the address;
C. had the addressD. had had the address;
E. had had the address,
16. Do you know that these gloves have lay on the bureau all week?
A. have lay on
B. have laid on
C. would lie on
D. had laid on
E. have lain on
17. If I would have known about the team tryouts, I would have signed up for them.
A. would have known
B. would had known
C. could of known
D. had been told
E. could have been told
18. If he would have revised his first draft, he would have received a better grade.
A. would have revised
B. had revised
C. could of revised
D. had of revised
E. would revise
19. Valarie claims that cats made the best pets.
A. made the best pets.
B. could be the best pets.
C. are the best pets.
D. make of the best pets
E. make the best pets.
20. By next month Ms. Jones will be Mayor of Tallahassee for two years.
A. will be Mayor of Tallahassee
B. will have been Mayor of Tallahassee
C. will be mayor of Tallahassee
D. will have been mayor of Tallahassee
E. could have been mayor of Tallahassee
Answer Key
1. D
2. D
3. A
4. D
5. C
6. B
7. D
8. D
9. D
10. E
11. C
12. A
13. B
14. D
15. E
16. E
17. A
18. B
19. E
20. D
Practice Questions: Sentence Correction 2
1. Hours of driving laid ahead of us.
A. laid
B. have lain
C. lay
D. has lay
E. lie
2. By the time we get to the picnic area, the rain will stop.
A. will stop
B. shall stop
C. will has stopped
D. shall have stopped
E. will have stopped
3. If Judy would not have missed the deadline, the yearbook delivery would have been on time.
A. would not have missed
B. should have not missed
C. wouldn't have missed
D. had not missed
E. would have not missed
4. We spent Sunday afternoon wandering aimless in the park.
A. wandering aimless
B. wandering aimlessly
C. wandering without purpose
D. wandering in an aimless manner
E. wandering almost aimlessly
5. Only after I went home did I remember my dental appointment.
A. went home
B. had went home
C. had gone home
D. gone home
E. should go home
6. The book lay open at page 77.
A. lay open
B. laid open
C. lied open
D. lain open
E. was laid open
7. By this time next year Johanna will begin classes at the University of Colorado.
A. will begin classes
B. will have begun classes
C. has began classes
D. should begin classes
E. should have begun classes
8. After comparing my air conditioner with the one on sale, I decided that mine was the most efficient.
A. was the most efficient.
B. should be the most efficient.
C. was the more efficient.
D. was, by far the most efficient
E. should be considered the most efficient.
9. I would have liked to have gone swimming yesterday.
A. to have gone swimming
B. to go swimming
C. to had gone swimming
D. to go to swim
E. to of gone swimming
10. I wish I read the chapter before I tried to answer the questions.
A. read the chapter
B. would read the chapter
C. should of read the chapter
D. could have read the chapter
E. had read the chapter
11. Nathanael West said that he'd never have written his satirical novel if he had not visited Hollywood.
A. have written his
B. would have written his
C. could of written his
D. could have written his
E. should of written his
12. The smell from the paper mill laid over the town like a blanket.
A. laid
B. has lain
C. will lie
D. lay
E. has laid
13. When I was halfway down the stairs, I suddenly knew what I had wanted to have said.
A. to have said
B. too say
C. to have been said
D. to had say
E. to say
14. I would be more careful if I had been you.
A. had been
B. could have been
C. was
D. were
E. could have been
15. They read where the governor has appointed a special committee to improve the school calendar.
A. where
B. how
C. were
D. of where
E. wear
16. In study hall I sit besides Paul Smith, who is captain of the swim team and one of the best swimmers in the
state.
A. sit besides
B. sat beside
C. have set beside
D. sit beside
E. have sit beside
17. This classic has been read with enjoyment for nearly two hundred years.
A. has been read
B. will have been read
C. shall have been read
D. is being read
E. was read
18. Many nineteenth-century biographers rely on their imagination, not on real facts.
A. rely on their imagination,
B. relied on their imagination,
C. have relied on their imagination
D. could have relied on their imagination,
E. could rely on their imaginations:
19. The private lives of politicians, generals, and other notables fascinates the reading public.
A. fascinates the reading
B. have fascinated the reading
C. will fascinate the reading
D. fascinate the reading
E. has fascinate the reading
20. That small man chose a seat near the door and carefully sat down.
A. sat
B. will sit
C. could of sat
D. have sit down
E. set down
21. Last summer I worked in the chemical laboratory at the Brass Company; most the work came into the lab
for testing marked with the words top priority.
A. words top priority.
B. words - top priority.
C. words: Top priority.
D. words, “Top Priority.”
E. Words “top priority.”
Answer Key
1. C
2. E
3. D
4. B
5. C
6. A
7. B
8. C
9. A
10. E
11. A
12. D
13. E
14. D
15. A
16. D
17. A
18. B
19. D
20. A
21. D